squiggle.c/references/how-to-calculate-the-inverse-of-the-normal-cumulative-distribution-function-in-p.1
2023-12-03 18:46:24 +00:00

2499 lines
186 KiB
Groff
Raw Blame History

This file contains ambiguous Unicode characters

This file contains Unicode characters that might be confused with other characters. If you think that this is intentional, you can safely ignore this warning. Use the Escape button to reveal them.

<!DOCTYPE html>
<html itemscope itemtype="https://schema.org/QAPage" class="html__responsive " lang="en">
<head>
<title>scipy - How to calculate the inverse of the normal cumulative distribution function in python? - Stack Overflow</title>
<link rel="shortcut icon" href="https://cdn.sstatic.net/Sites/stackoverflow/Img/favicon.ico?v=ec617d715196">
<link rel="apple-touch-icon" href="https://cdn.sstatic.net/Sites/stackoverflow/Img/apple-touch-icon.png?v=c78bd457575a">
<link rel="image_src" href="https://cdn.sstatic.net/Sites/stackoverflow/Img/apple-touch-icon.png?v=c78bd457575a">
<link rel="search" type="application/opensearchdescription+xml" title="Stack Overflow" href="/opensearch.xml">
<link rel="canonical" href="https://stackoverflow.com/questions/20626994/how-to-calculate-the-inverse-of-the-normal-cumulative-distribution-function-in-p" />
<meta name="viewport" content="width=device-width, height=device-height, initial-scale=1.0, minimum-scale=1.0">
<meta property="og:type" content= "website" />
<meta property="og:url" content="https://stackoverflow.com/questions/20626994/how-to-calculate-the-inverse-of-the-normal-cumulative-distribution-function-in-p"/>
<meta property="og:site_name" content="Stack Overflow" />
<meta property="og:image" itemprop="image primaryImageOfPage" content="https://cdn.sstatic.net/Sites/stackoverflow/Img/apple-touch-icon@2.png?v=73d79a89bded" />
<meta name="twitter:card" content="summary"/>
<meta name="twitter:domain" content="stackoverflow.com"/>
<meta name="twitter:title" property="og:title" itemprop="name" content="How to calculate the inverse of the normal cumulative distribution function in python?" />
<meta name="twitter:description" property="og:description" itemprop="description" content="How do I calculate the inverse of the cumulative distribution function (CDF) of the normal distribution in Python?&#xA;&#xA;Which library should I use? Possibly scipy?" />
<script id="webpack-public-path" type="text/uri-list">https://cdn.sstatic.net/</script>
<script src="https://ajax.googleapis.com/ajax/libs/jquery/1.12.4/jquery.min.js"></script>
<script defer src="https://cdn.sstatic.net/Js/third-party/npm/@stackoverflow/stacks/dist/js/stacks.min.js?v=d5f780ae3281"></script>
<script src="https://cdn.sstatic.net/Js/stub.en.js?v=fa8bcab580cd"></script>
<link rel="stylesheet" type="text/css" href="https://cdn.sstatic.net/Shared/stacks.css?v=5d83236da4e2">
<link rel="stylesheet" type="text/css" href="https://cdn.sstatic.net/Sites/stackoverflow/primary.css?v=89d00f6d5eec">
<link rel="alternate" type="application/atom+xml" title="Feed for question &#x27;How to calculate the inverse of the normal cumulative distribution function in python?&#x27;" href="/feeds/question/20626994">
<script>
StackExchange.ready(function () {
StackExchange.using("snippets", function () {
StackExchange.snippets.initSnippetRenderer();
});
StackExchange.using("postValidation", function () {
StackExchange.postValidation.initOnBlurAndSubmit($('#post-form'), 2, 'answer');
});
StackExchange.question.init({showAnswerHelp:true,showTrendingSortLaunchPopover:false,showTrendingSortPostLaunchPopover:false,totalCommentCount:3,shownCommentCount:3,enableTables:true,questionId:20626994});
styleCode();
StackExchange.realtime.subscribeToQuestion('1', '20626994');
StackExchange.using("gps", function () { StackExchange.gps.trackOutboundClicks('#content', '.js-post-body'); });
});
</script>
<link rel="stylesheet" type="text/css" href="https://cdn.sstatic.net/Shared/Channels/channels.css?v=64256d36becc">
<script type="application/json" data-role="module-args" data-module-name="Shared/options.mod">{"options":{"locale":"en","serverTime":1701628369,"routeName":"Questions/Show","stackAuthUrl":"https://stackauth.com","networkMetaHostname":"meta.stackexchange.com","site":{"name":"Stack Overflow","description":"Q\u0026A for professional and enthusiast programmers","isNoticesTabEnabled":true,"enableNewTagCreationWarning":true,"insertSpaceAfterNameTabCompletion":false,"id":1,"cookieDomain":".stackoverflow.com","childUrl":"https://meta.stackoverflow.com","negativeVoteScoreFloor":null,"enableSocialMediaInSharePopup":true,"protocol":"https"},"user":{"fkey":"d8a1f9f0d8241e1b245c6f93920897c7b1457a3e09f1f846dafb3a27efbe14a3","tid":"3de1ec46-61f8-4c2b-8c1d-22354c9bb232","rep":0,"isAnonymous":true,"isAnonymousNetworkWide":true,"ab":{"job_search_status":{"v":"ai_policy_banner_v1","g":1}}},"events":{"postType":{"question":1},"postEditionSection":{"title":1,"body":2,"tags":3}}}}</script>
<script type="application/json" data-role="module-args" data-module-name="Shared/settings.mod">{"settings":{"userMessaging":{"showNewFeatureNotice":true},"tags":{},"subscriptions":{"defaultBasicMaxTrueUpSeats":250,"defaultFreemiumMaxTrueUpSeats":50,"defaultMaxTrueUpSeats":1000},"snippets":{"renderDomain":"stacksnippets.net","snippetsEnabled":true},"site":{"allowImageUploads":true,"enableImgurHttps":true,"enableUserHovercards":true,"forceHttpsImages":true,"stacksEditorPreviewEnabled":true,"styleCode":true},"search":{},"questions":{"enableQuestionTitleLengthLiveWarning":true,"enableSavesFeature":true,"maxTitleSize":150,"questionTitleLengthStartLiveWarningChars":50},"intercom":{"appId":"inf0secd","enableJavascriptImplementationFor":true,"hostBaseUrl":"https://stacksnippets.net"},"paths":{"jQueryUICSSPath":"https://ajax.googleapis.com/ajax/libs/jqueryui/1.12.0/themes/smoothness/jquery-ui.css","jQueryUIJSPath":"https://ajax.googleapis.com/ajax/libs/jqueryui/1.12.0/jquery-ui.min.js"},"mentions":{"maxNumUsersInDropdown":50},"markdown":{"enableTables":true},"legal":{"oneTrustConfigId":"c3d9f1e3-55f3-4eba-b268-46cee4c6789c"},"flags":{"allowRetractingCommentFlags":true,"allowRetractingFlags":true},"elections":{"opaVoteResultsBaseUrl":"https://www.opavote.com/results/"},"comments":{},"accounts":{"currentPasswordRequiredForChangingStackIdPassword":true}}}</script>
<script>StackExchange.init();</script>
<script>
StackExchange.using.setCacheBreakers({"Js/adops.en.js":"6da43f5e0a84","Js/ask.en.js":"","Js/begin-edit-event.en.js":"20edbaccceae","Js/copy-transpiled.en.js":"d31dc7eba3bc","Js/events.en.js":"","Js/explore-qlist.en.js":"2b1f34938b8b","Js/full-anon.en.js":"192ecaea0c9d","Js/full.en.js":"efb11489a333","Js/highlightjs-loader.en.js":"510e2f94c2bf","Js/inline-tag-editing.en.js":"3e8cc64ee9d6","Js/keyboard-shortcuts.en.js":"107c2ac31497","Js/markdown-it-loader.en.js":"5818ef89ff9d","Js/mentions-transpiled.en.js":"36b8cabd2c12","Js/moderator.en.js":"bd04c908e875","Js/postCollections-transpiled.en.js":"ea1228154a76","Js/post-validation.en.js":"8fd6c83cea6c","Js/question-editor.en.js":"","Js/review-v2-transpiled.en.js":"7b6b513b5808","Js/revisions.en.js":"47b4d5ac24c9","Js/stacks-editor.en.js":"45ddc00eb16e","Js/tageditor.en.js":"dc13482a67f8","Js/tageditornew.en.js":"b11be3ff22c6","Js/tagsuggestions.en.js":"bd6ec908f2a7","Js/unlimited-transpiled.en.js":"f26a1d5f3365","Js/wmd.en.js":"033a0412fcae","Js/snippet-javascript-codemirror.en.js":"ae1dcf38deb7"});
StackExchange.using("gps", function() {
StackExchange.gps.init(false);
});
</script>
<noscript id="noscript-css"><style>body,.s-topbar{margin-top:1.9em}</style></noscript>
</head>
<body class="question-page unified-theme">
<div id="notify-container"></div>
<div id="custom-header"></div>
<header class="s-topbar ps-fixed t0 l0 js-top-bar">
<div class="s-topbar--container">
<a href="#" class="s-topbar--menu-btn js-left-sidebar-toggle" role="menuitem" aria-haspopup="true" aria-controls="left-sidebar" aria-expanded="false"><span></span></a>
<div class="topbar-dialog leftnav-dialog js-leftnav-dialog dno">
<div class="left-sidebar js-unpinned-left-sidebar" data-can-be="left-sidebar" data-is-here-when="sm"></div>
</div>
<a href="https://stackoverflow.com" class="s-topbar--logo js-gps-track"
data-gps-track="top_nav.click({is_current:false, location:2, destination:8})">
<span class="-img _glyph">Stack Overflow</span>
</a>
<ol class="s-navigation" role="presentation">
<li class="md:d-none">
<a href="https://stackoverflow.co/" class="s-navigation--item js-gps-track"
data-gps-track="top_nav.products.click({location:2, destination:7})"
data-ga="[&quot;top navigation&quot;,&quot;about menu click&quot;,null,null,null]">About</a>
</li>
<li>
<a href="#"
class="s-navigation--item js-gps-track js-products-menu"
aria-controls="products-popover"
data-controller="s-popover"
data-action="s-popover#toggle"
data-s-popover-placement="bottom"
data-s-popover-toggle-class="is-selected"
data-gps-track="top_nav.products.click({location:2, destination:1})"
data-ga="[&quot;top navigation&quot;,&quot;products menu click&quot;,null,null,null]">
Products
</a>
</li>
<li class="md:d-none">
<a href="https://stackoverflow.co/teams/" class="s-navigation--item js-gps-track"
data-gps-track="top_nav.products.click({location:2, destination:7})"
data-ga="[&quot;top navigation&quot;,&quot;learn more - teams&quot;,null,null,null]">For Teams</a>
</li>
</ol>
<div class="s-popover ws2 mtn2 p0"
id="products-popover"
role="menu"
aria-hidden="true">
<div class="s-popover--arrow"></div>
<ol class="list-reset s-anchors s-anchors__inherit">
<li class="m6">
<a href="/questions" class="bar-sm p6 d-block h:bg-black-225 js-gps-track"
data-gps-track="top_nav.products.click({location:2, destination:2})"
data-ga="[&quot;top navigation&quot;,&quot;public qa submenu click&quot;,null,null,null]">
<span class="fs-body1 d-block">Stack Overflow</span>
<span class="fs-caption d-block fc-black-400">Public questions &amp; answers</span>
</a>
</li>
<li class="m6">
<a href="https://stackoverflow.co/teams/" class="bar-sm p6 d-block h:bg-black-225 js-gps-track"
data-gps-track="top_nav.products.click({location:2, destination:3})"
data-ga="[&quot;top navigation&quot;,&quot;teams submenu click&quot;,null,null,null]">
<span class="fs-body1 d-block">Stack Overflow for Teams</span>
<span class="fs-caption d-block fc-black-400">Where developers &amp; technologists share private knowledge with coworkers</span>
</a>
</li>
<li class="m6">
<a href="https://stackoverflow.co/talent/" class="bar-sm p6 d-block h:bg-black-225 js-gps-track"
data-gps-track="top_nav.products.click({location:2, destination:5})"
data-ga="[&quot;top navigation&quot;,&quot;talent submenu click&quot;,null,null,null]">
<span class="fs-body1 d-block">Talent</span>
<span class="fs-caption d-block fc-black-400">
Build your employer brand
</span>
</a>
</li>
<li class="m6">
<a href="https://stackoverflow.co/advertising/" class="bar-sm p6 d-block h:bg-black-225 js-gps-track"
data-gps-track="top_nav.products.click({location:2, destination:6})"
data-ga="[&quot;top navigation&quot;,&quot;advertising submenu click&quot;,null,null,null]">
<span class="fs-body1 d-block">Advertising</span>
<span class="fs-caption d-block fc-black-400">Reach developers &amp; technologists worldwide</span>
</a>
</li>
<li class="bt bc-black-200 py6 px6 bbr-md">
<a href="https://stackoverflow.co/labs/" class="bar-sm p6 d-block h:bg-black-225 js-gps-track"
data-gps-track="top_nav.products.click({location:2, destination:7})"
data-ga="[&quot;top navigation&quot;,&quot;labs submenu click&quot;,null,null,null]">
<span class="fs-body1 d-block">Labs</span>
<span class="fs-caption d-block fc-black-400">The future of collective knowledge sharing</span>
</a>
</li>
<li class="bg-black-100 bt bc-black-200 py6 px6 bbr-md">
<a href="https://stackoverflow.co/" class="fc-black-400 d-block py6 px6 h:fc-black-600 js-gps-track"
data-gps-track="top_nav.products.click({location:2, destination:7})"
data-ga="[&quot;top navigation&quot;,&quot;about submenu click&quot;,null,null,null]">About the company</a>
</li>
</ol>
</div>
<form id="search" role="search" action=/search class="s-topbar--searchbar js-searchbar " autocomplete="off">
<div class="s-topbar--searchbar--input-group">
<input name="q"
type="text"
role="combobox"
placeholder="Search&#x2026;"
value=""
autocomplete="off"
maxlength="240"
class="s-input s-input__search js-search-field "
aria-label="Search"
aria-controls="top-search"
data-controller="s-popover"
data-action="focus->s-popover#show"
data-s-popover-placement="bottom-start" />
<svg aria-hidden="true" class="s-input-icon s-input-icon__search svg-icon iconSearch" width="18" height="18" viewBox="0 0 18 18"><path d="m18 16.5-5.14-5.18h-.35a7 7 0 1 0-1.19 1.19v.35L16.5 18l1.5-1.5ZM12 7A5 5 0 1 1 2 7a5 5 0 0 1 10 0Z"/></svg>
<div class="s-popover p0 wmx100 wmn4 sm:wmn-initial js-top-search-popover" id="top-search" role="menu">
<div class="s-popover--arrow"></div>
<div class="js-spinner p24 d-flex ai-center jc-center d-none">
<div class="s-spinner s-spinner__sm fc-orange-400">
<div class="v-visible-sr">Loading&#x2026;</div>
</div>
</div>
<span class="v-visible-sr js-screen-reader-info"></span>
<div class="js-ac-results overflow-y-auto hmx3 d-none"></div>
<div class="js-search-hints" aria-describedby="Tips for searching"></div>
</div>
</div>
</form>
<nav class="h100 ml-auto overflow-x-auto pr12">
<ol class="s-topbar--content" role="menubar">
<li class="js-topbar-dialog-corral" role="presentation">
<div class="topbar-dialog siteSwitcher-dialog dno" role="menu">
<div class="header fw-wrap">
<h3 class="flex--item">
<a href="https://stackoverflow.com">current community</a>
</h3>
<div class="flex--item fl1">
<div class="ai-center d-flex jc-end">
<button
class="js-close-button s-btn s-btn__muted p0 ml8 d-none sm:d-block"
type="button"
aria-label="Close"
>
<svg aria-hidden="true" class="svg-icon iconClear" width="18" height="18" viewBox="0 0 18 18"><path d="M15 4.41 13.59 3 9 7.59 4.41 3 3 4.41 7.59 9 3 13.59 4.41 15 9 10.41 13.59 15 15 13.59 10.41 9 15 4.41Z"/></svg>
</button>
</div>
</div>
</div>
<div class="modal-content bg-blue-200 current-site-container">
<ul class="current-site ">
<li class="d-flex">
<div class="fl1">
<a href="https://stackoverflow.com"
class="current-site-link site-link js-gps-track d-flex gs8 gsx"
data-id="1"
data-gps-track="site_switcher.click({ item_type:3 })">
<div class="favicon favicon-stackoverflow site-icon flex--item" title="Stack Overflow"></div>
<span class="flex--item fl1">
Stack Overflow
</span>
</a>
</div>
<div class="related-links">
<a href="https://stackoverflow.com/help" class="js-gps-track" data-gps-track="site_switcher.click({ item_type:14 })">help</a>
<a href="https://chat.stackoverflow.com/?tab=site&amp;host=stackoverflow.com" class="js-gps-track" data-gps-track="site_switcher.click({ item_type:6 })">chat</a>
</div>
</li>
<li class="related-site d-flex">
<div class="L-shaped-icon-container">
<span class="L-shaped-icon"></span>
</div>
<a href="https://meta.stackoverflow.com"
class=" site-link js-gps-track d-flex gs8 gsx"
data-id="552"
data-gps-track="site.switch({ target_site:552, item_type:3 }),site_switcher.click({ item_type:4 })">
<div class="favicon favicon-stackoverflowmeta site-icon flex--item" title="Meta Stack Overflow"></div>
<span class="flex--item fl1">
Meta Stack Overflow
</span>
</a>
</li>
</ul>
</div>
<div class="header" id="your-communities-header">
<h3>
your communities </h3>
</div>
<div class="modal-content" id="your-communities-section">
<div class="call-to-login">
<a href="https://stackoverflow.com/users/signup?ssrc=site_switcher&amp;returnurl=https%3a%2f%2fstackoverflow.com%2fquestions%2f20626994%2fhow-to-calculate-the-inverse-of-the-normal-cumulative-distribution-function-in-p" class="login-link js-gps-track" data-gps-track="site_switcher.click({ item_type:10 })">Sign up</a> or <a href="https://stackoverflow.com/users/login?ssrc=site_switcher&amp;returnurl=https%3a%2f%2fstackoverflow.com%2fquestions%2f20626994%2fhow-to-calculate-the-inverse-of-the-normal-cumulative-distribution-function-in-p" class="login-link js-gps-track" data-gps-track="site_switcher.click({ item_type:11 })">log in</a> to customize your list. </div>
</div>
<div class="header">
<h3><a href="https://stackexchange.com/sites">more stack exchange communities</a>
</h3>
<a href="https://stackoverflow.blog" class="float-right">company blog</a>
</div>
<div class="modal-content">
<div class="child-content"></div>
</div>
</div>
</li>
<li role="none"><button class="s-topbar--item s-btn s-btn__icon s-btn__muted d-none sm:d-inline-flex js-searchbar-trigger" role="menuitem" aria-label="Search" aria-haspopup="true" aria-controls="search" title="Click to show search"><svg aria-hidden="true" class="svg-icon iconSearch" width="18" height="18" viewBox="0 0 18 18"><path d="m18 16.5-5.14-5.18h-.35a7 7 0 1 0-1.19 1.19v.35L16.5 18l1.5-1.5ZM12 7A5 5 0 1 1 2 7a5 5 0 0 1 10 0Z"/></svg></button></li>
<li role="none">
<a href="https://stackoverflow.com/users/login?ssrc=head&returnurl=https%3a%2f%2fstackoverflow.com%2fquestions%2f20626994%2fhow-to-calculate-the-inverse-of-the-normal-cumulative-distribution-function-in-p" class="s-topbar--item s-topbar--item__unset s-btn s-btn__outlined ws-nowrap js-gps-track" role="menuitem" rel="nofollow"
data-gps-track="login.click" data-ga="[&quot;top navigation&quot;,&quot;login button click&quot;,null,null,null]">Log in</a>
</li>
<li role="none"><a href="https://stackoverflow.com/users/signup?ssrc=head&returnurl=https%3a%2f%2fstackoverflow.com%2fquestions%2f20626994%2fhow-to-calculate-the-inverse-of-the-normal-cumulative-distribution-function-in-p" class="s-topbar--item s-topbar--item__unset ml4 s-btn s-btn__filled ws-nowrap" role="menuitem" rel="nofollow" data-ga="[&quot;sign up&quot;,&quot;Sign Up Navigation&quot;,&quot;Header&quot;,null,null]">Sign up</a></li>
</ol>
</nav>
</div>
</header>
<script>
StackExchange.ready(function () { StackExchange.topbar.init(); });
StackExchange.scrollPadding.setPaddingTop(50, 10);
</script>
<div class="container">
<div id="left-sidebar" data-is-here-when="md lg" class="left-sidebar js-pinned-left-sidebar ps-relative">
<div class="left-sidebar--sticky-container js-sticky-leftnav">
<nav role="navigation">
<ol class="nav-links">
<li>
<ol class="nav-links">
<li class="ps-relative" aria-current="false">
<a
href="/"
class="pl8 js-gps-track nav-links--link -link__with-icon"
data-gps-track="top_nav.click({is_current: false, location:2, destination:8, has_activity_notification:False})"
aria-controls="" data-controller="" data-s-popover-placement="right"
aria-current="false"
data-s-popover-auto-show="true" data-s-popover-hide-on-outside-click="never"
>
<svg aria-hidden="true" class="svg-icon iconHome" width="18" height="18" viewBox="0 0 18 18"><path d="M15 10v5a2 2 0 0 1-2 2H5a2 2 0 0 1-2-2v-5H0l9-9 9 9h-3Zm-8 1v6h4v-6H7Z"/></svg> <span class="-link--channel-name pl6">Home</span>
</a>
</li>
<li class="ps-relative youarehere" aria-current="true">
<a id="nav-questions"
href="/questions"
class="pl8 js-gps-track nav-links--link -link__with-icon"
data-gps-track="top_nav.click({is_current: true, location:2, destination:1, has_activity_notification:False})"
aria-controls="" data-controller="" data-s-popover-placement="right"
aria-current="false"
data-s-popover-auto-show="true" data-s-popover-hide-on-outside-click="never"
>
<svg aria-hidden="true" class="svg-icon iconQuestion" width="18" height="18" viewBox="0 0 18 18"><path d="m4 15-3 3V4c0-1.1.9-2 2-2h12c1.09 0 2 .91 2 2v9c0 1.09-.91 2-2 2H4Zm7.75-3.97c.72-.83.98-1.86.98-2.94 0-1.65-.7-3.22-2.3-3.83a4.41 4.41 0 0 0-3.02 0 3.8 3.8 0 0 0-2.32 3.83c0 1.29.35 2.29 1.03 3a3.8 3.8 0 0 0 2.85 1.07c.62 0 1.2-.11 1.71-.34.65.44 1 .68 1.06.7.23.13.46.23.7.3l.59-1.13a5.2 5.2 0 0 1-1.28-.66Zm-1.27-.9a5.4 5.4 0 0 0-1.5-.8l-.45.9c.33.12.66.29.98.5-.2.07-.42.11-.65.11-.61 0-1.12-.23-1.52-.68-.86-1-.86-3.12 0-4.11.8-.9 2.35-.9 3.15 0 .9 1.01.86 3.03-.01 4.08Z"/></svg> <span class="-link--channel-name pl6">Questions</span>
</a>
</li>
<li class="ps-relative" aria-current="false">
<a
href="/tags"
class="pl8 js-gps-track nav-links--link -link__with-icon"
data-gps-track="top_nav.click({is_current: false, location:2, destination:2, has_activity_notification:False})"
aria-controls="" data-controller="" data-s-popover-placement="right"
aria-current="false"
data-s-popover-auto-show="true" data-s-popover-hide-on-outside-click="never"
>
<svg aria-hidden="true" class="svg-icon iconTags" width="18" height="18" viewBox="0 0 18 18"><path d="M9.24 1a3 3 0 0 0-2.12.88l-5.7 5.7a2 2 0 0 0-.38 2.31 3 3 0 0 1 .67-1.01l6-6A3 3 0 0 1 9.83 2H14a3 3 0 0 1 .79.1A2 2 0 0 0 13 1H9.24Z" opacity=".4"/><path d="M9.83 3a2 2 0 0 0-1.42.59l-6 6a2 2 0 0 0 0 2.82L6.6 16.6a2 2 0 0 0 2.82 0l6-6A2 2 0 0 0 16 9.17V5a2 2 0 0 0-2-2H9.83ZM12 9a2 2 0 1 1 0-4 2 2 0 0 1 0 4Z"/></svg> <span class="-link--channel-name pl6">Tags</span>
</a>
</li>
<li class="pb24"></li>
<li class="ps-relative" aria-current="false">
<a id="nav-users"
href="/users"
class="pl8 js-gps-track nav-links--link -link__with-icon"
data-gps-track="top_nav.click({is_current: false, location:2, destination:3, has_activity_notification:False})"
aria-controls="" data-controller="" data-s-popover-placement="right"
aria-current="false"
data-s-popover-auto-show="true" data-s-popover-hide-on-outside-click="never"
>
<svg aria-hidden="true" class="svg-icon iconPeople" width="18" height="18" viewBox="0 0 18 18"><path d="M17 14c0 .44-.45 1-1 1H9a1 1 0 0 1-1-1H2c-.54 0-1-.56-1-1 0-2.63 3-4 3-4s.23-.4 0-1c-.84-.62-1.06-.59-1-3 .06-2.42 1.37-3 2.5-3s2.44.58 2.5 3c.06 2.41-.16 2.38-1 3-.23.59 0 1 0 1s1.55.71 2.42 2.09c.78-.72 1.58-1.1 1.58-1.1s.23-.4 0-1c-.84-.61-1.06-.58-1-3 .06-2.41 1.37-3 2.5-3s2.44.59 2.5 3c.05 2.42-.16 2.39-1 3-.23.6 0 1 0 1s3 1.38 3 4Z"/></svg> <span class="-link--channel-name pl6">Users</span>
</a>
</li>
<li class="ps-relative" aria-current="false">
<a id="nav-companies"
href="https://stackoverflow.com/jobs/companies?so_medium=stackoverflow&amp;so_source=SiteNav"
class="pl8 js-gps-track nav-links--link -link__with-icon"
data-gps-track="top_nav.click({is_current: false, location:2, destination:12, has_activity_notification:False})"
aria-controls="" data-controller="" data-s-popover-placement="right"
aria-current="false"
data-s-popover-auto-show="true" data-s-popover-hide-on-outside-click="never"
>
<svg aria-hidden="true" class="svg-icon iconBriefcase" width="18" height="18" viewBox="0 0 18 18"><path d="M5 4a1 1 0 0 1 1-1h6a1 1 0 0 1 1 1v1h1a2 2 0 0 1 2 2v6a2 2 0 0 1-2 2H4a2 2 0 0 1-2-2V7c0-1.1.9-2 2-2h1V4Zm7 0H6v1h6V4Z"/></svg> <span class="-link--channel-name pl6">Companies</span>
</a>
</li>
<li class="ml8 mt32 mb8">
<div class="d-flex jc-space-between ai-center">
<a
class="s-link d-flex fl-grow1 fc-black-400 h:fc-black-600 fs-fine js-collectives-navcta-toggle"
href="javascript:void(0)"
role="button"
aria-controls="popover-discover-collectives"
data-controller="s-popover"
data-action="s-popover#toggle"
data-s-popover-placement="top"
data-s-popover-toggle-class="is-selected"
data-gps-track="top_nav.click({is_current:false, location:2, destination:17})"
>
<div class="flex--item fl-grow1 tt-uppercase fc-black-600 fw-bold">Collectives</div>
<div class="flex--item px12">
<svg aria-hidden="true" class="svg-icon iconPlusSm" width="14" height="14" viewBox="0 0 14 14"><path d="M8 2H6v4H2v2h4v4h2V8h4V6H8V2Z"/></svg> </div>
</a>
</div>
</li>
<li class="ps-relative" aria-current="false">
<a id="nav-collective-discover"
href="/collectives"
class="pl8 ai-center js-collectives-navcta-toggle js-gps-track nav-links--link -link__with-icon"
data-gps-track="top_nav.click({is_current: false, location:2, destination:18, has_activity_notification:False})"
aria-controls="" data-controller="" data-s-popover-placement="right"
aria-current="false"
data-s-popover-auto-show="true" data-s-popover-hide-on-outside-click="never"
>
<svg aria-hidden="true" class="mt-auto fc-orange-400 svg-icon iconStarVerified" width="18" height="18" viewBox="0 0 18 18"><path d="M9.86.89a1.14 1.14 0 0 0-1.72 0l-.5.58c-.3.35-.79.48-1.23.33l-.72-.25a1.14 1.14 0 0 0-1.49.85l-.14.76c-.1.45-.45.8-.9.9l-.76.14c-.67.14-1.08.83-.85 1.49l.25.72c.15.44.02.92-.33 1.23l-.58.5a1.14 1.14 0 0 0 0 1.72l.58.5c.35.3.48.79.33 1.23l-.25.72c-.23.66.18 1.35.85 1.49l.76.14c.45.1.8.45.9.9l.14.76c.14.67.83 1.08 1.49.85l.72-.25c.44-.15.92-.02 1.23.33l.5.58c.46.52 1.26.52 1.72 0l.5-.58c.3-.35.79-.48 1.23-.33l.72.25c.66.23 1.35-.18 1.49-.85l.14-.76c.1-.45.45-.8.9-.9l.76-.14c.67-.14 1.08-.83.85-1.49l-.25-.72c-.15-.44-.02-.92.33-1.23l.58-.5c.52-.46.52-1.26 0-1.72l-.58-.5c-.35-.3-.48-.79-.33-1.23l.25-.72a1.14 1.14 0 0 0-.85-1.49l-.76-.14c-.45-.1-.8-.45-.9-.9l-.14-.76a1.14 1.14 0 0 0-1.49-.85l-.72.25c-.44.15-.92.02-1.23-.33l-.5-.58Zm-.49 2.67L10.6 6.6c.05.15.19.24.34.25l3.26.22c.36.03.5.48.23.71l-2.5 2.1a.4.4 0 0 0-.14.4l.8 3.16a.4.4 0 0 1-.6.44L9.2 12.13a.4.4 0 0 0-.42 0l-2.77 1.74a.4.4 0 0 1-.6-.44l.8-3.16a.4.4 0 0 0-.13-.4l-2.5-2.1a.4.4 0 0 1 .22-.7l3.26-.23a.4.4 0 0 0 .34-.25l1.22-3.03a.4.4 0 0 1 .74 0Z"/></svg> <span class="-link--channel-name pl6">Explore Collectives</span>
</a>
</li>
<li class="ml8 mt32 mb8">
<a href="javascript:void(0)"
class="s-link s-link d-flex fl-grow1 fc-black-400 h:fc-black-600 fs-fine"
role="button"
aria-controls="popover-labs-left-nav"
data-controller="s-popover"
data-action="s-popover#toggle"
data-s-popover-placement="top"
data-s-popover-toggle-class="is-selected"
>
<div class="flex--item fl-grow1 tt-uppercase fc-black-600 fw-bold">Labs</div>
<div class="flex--item px12">
<svg aria-hidden="true" class="svg-icon iconInfoSm" width="14" height="14" viewBox="0 0 14 14"><path d="M7 1a6 6 0 1 1 0 12A6 6 0 0 1 7 1Zm1 10V6H6v5h2Zm0-6V3H6v2h2Z"/></svg>
</div>
</a>
</li>
<li class="ps-relative" aria-current="false">
<a id="nav-labs-discussions"
href="/collectives/beta/discussions"
class="pl8 ai-center js-gps-track nav-links--link -link__with-icon"
data-gps-track="top_nav.click({is_current: false, location:2, destination:24, has_activity_notification:False})"
aria-controls="" data-controller="" data-s-popover-placement="right"
aria-current="false"
data-s-popover-auto-show="true" data-s-popover-hide-on-outside-click="never"
>
<svg aria-hidden="true" class="fc-black-400 w16 svg-icon iconSpeechBubble" width="18" height="18" viewBox="0 0 18 18"><path d="m4 15-3 3V4c0-1.1.9-2 2-2h12a2 2 0 0 1 2 2v9a2 2 0 0 1-2 2H4Z"/></svg> <span class="-link--channel-name pl6">Discussions</span>
</a>
</li>
</ol>
</li>
<li class="js-freemium-cta ps-relative">
<div class="fs-fine tt-uppercase fc-black-600 fw-bold ml8 mt16 mb8">Teams</div>
<div class="bt bl bb bc-black-200 p12 pb6 fc-black-500 blr-sm overflow-hidden">
<strong class="fc-black-600 mb6">Stack Overflow for Teams</strong>
Start collaborating and sharing organizational knowledge.
<img class="wmx100 mx-auto my8 h-auto d-block" width="139" height="114" src="https://cdn.sstatic.net/Img/teams/teams-illo-free-sidebar-promo.svg?v=47faa659a05e" alt="">
<a href="https://try.stackoverflow.co/why-teams/?utm_source=so-owned&amp;utm_medium=side-bar&amp;utm_campaign=campaign-38&amp;utm_content=cta"
class="w100 s-btn s-btn__filled s-btn__xs bg-orange-400 js-gps-track"
data-gps-track="teams.create.left-sidenav.click({ Action: 6 })"
data-ga="[&quot;teams left navigation - anonymous&quot;,&quot;left nav free cta&quot;,&quot;stackoverflow.com/teams/create/free&quot;,null,null]">Create a free Team</a>
<a href="https://stackoverflow.co/teams/"
class="w100 s-btn s-btn__muted s-btn__xs js-gps-track"
data-gps-track="teams.create.left-sidenav.click({ Action: 5 })"
data-ga="[&quot;teams left navigation - anonymous&quot;,&quot;left nav free cta&quot;,&quot;stackoverflow.com/teams&quot;,null,null]">Why Teams?</a>
</div>
</li>
<li class="d-flex ai-center jc-space-between ml8 mt32 mb8 js-create-team-cta d-none">
<a href="javascript:void(0)"
class="s-link d-flex fl-grow1 fc-black-400 h:fc-black-600 fs-fine js-gps-track"
role="button"
aria-controls="popover-teams-create-cta"
data-controller="s-popover"
data-action="s-popover#toggle"
data-s-popover-placement="bottom-start"
data-s-popover-toggle-class="is-selected"
data-gps-track="teams.create.left-sidenav.click({ Action: ShowInfo })"
data-ga="[&quot;teams left navigation - anonymous&quot;,&quot;left nav show teams info&quot;,null,null,null]"
>
<div class="flex--item fl-grow1 fc-black-600 fw-bold tt-uppercase">Teams</div>
<div class="flex--item px12">
<svg aria-hidden="true" class="svg-icon iconPlusSm" width="14" height="14" viewBox="0 0 14 14"><path d="M8 2H6v4H2v2h4v4h2V8h4V6H8V2Z"/></svg>
</div>
</a>
</li>
<li class="ps-relative js-create-team-cta d-none">
<a href="https://stackoverflowteams.com/teams/create/free/?utm_source=so-owned&amp;utm_medium=side-bar&amp;utm_campaign=campaign-38&amp;utm_content=cta"
class="pl8 js-gps-track nav-links--link"
title="Stack Overflow for Teams is a private, secure spot for your organization's questions and answers."
data-gps-track="teams.create.left-sidenav.click({ Action: FreemiumTeamsCreateClick })"
data-ga="[&quot;teams left navigation - anonymous&quot;,&quot;left nav team click&quot;,&quot;stackoverflow.com/teams/create/free&quot;,null,null]">
<div class="d-flex ai-center">
<div class="flex--item s-avatar va-middle bg-orange-400">
<div class="s-avatar--letter mtn1">
<svg aria-hidden="true" class="svg-icon iconBriefcaseSm" width="14" height="14" viewBox="0 0 14 14"><path d="M4 3a1 1 0 0 1 1-1h4a1 1 0 0 1 1 1v1h.5c.83 0 1.5.67 1.5 1.5v5c0 .83-.67 1.5-1.5 1.5h-7A1.5 1.5 0 0 1 2 10.5v-5C2 4.67 2.67 4 3.5 4H4V3Zm5 1V3H5v1h4Z"/></svg>
</div>
<svg aria-hidden="true" class="native s-avatar--badge svg-icon iconShieldXSm" width="9" height="10" viewBox="0 0 9 10"><path fill="var(--white)" d="M0 1.84 4.5 0 9 1.84v3.17C9 7.53 6.3 10 4.5 10 2.7 10 0 7.53 0 5.01V1.84Z"/><path fill="var(--black-400)" d="M1 2.5 4.5 1 8 2.5v2.51C8 7.34 5.34 9 4.5 9 3.65 9 1 7.34 1 5.01V2.5Zm2.98 3.02L3.2 7h2.6l-.78-1.48a.4.4 0 0 1 .15-.38c.34-.24.73-.7.73-1.14 0-.71-.5-1.23-1.41-1.23-.92 0-1.39.52-1.39 1.23 0 .44.4.9.73 1.14.12.08.18.23.15.38Z"/></svg>
</div>
<div class="flex--item pl6">
Create free Team
</div>
</div>
</a>
</li>
</ol>
</nav>
</div>
<div class="s-popover ws2" id="popover-discover-collectives" role="menu">
<div class="s-popover--arrow"></div>
<div>
<svg aria-hidden="true" class="fc-orange-400 float-right ml24 svg-spot spotCollective" width="48" height="48" viewBox="0 0 48 48"><path d="M25.5 7a2.5 2.5 0 1 0 0-5 2.5 2.5 0 0 0 0 5ZM14 18.25c0-.69.56-1.25 1.25-1.25h22.5c.69 0 1.25.56 1.25 1.25V37.5a1 1 0 0 1-1.6.8l-4.07-3.05a1.25 1.25 0 0 0-.75-.25H15.25c-.69 0-1.25-.56-1.25-1.25v-15.5ZM7 24.5a2.5 2.5 0 1 1-5 0 2.5 2.5 0 0 1 5 0ZM25.5 48a2.5 2.5 0 1 0 0-5 2.5 2.5 0 0 0 0 5ZM48 24.5a2.5 2.5 0 1 1-5 0 2.5 2.5 0 0 1 5 0Z" opacity=".2"/><path d="M21 3.5a3.5 3.5 0 1 1 7 0 3.5 3.5 0 0 1-7 0ZM24.5 2a1.5 1.5 0 1 0 0 3 1.5 1.5 0 0 0 0-3ZM0 23.5a3.5 3.5 0 1 1 7 0 3.5 3.5 0 0 1-7 0ZM3.5 22a1.5 1.5 0 1 0 0 3 1.5 1.5 0 0 0 0-3ZM21 44.5a3.5 3.5 0 1 1 7 0 3.5 3.5 0 0 1-7 0Zm3.5-1.5a1.5 1.5 0 1 0 0 3 1.5 1.5 0 0 0 0-3Zm20-23a3.5 3.5 0 1 0 0 7 3.5 3.5 0 0 0 0-7ZM43 23.5a1.5 1.5 0 1 1 3 0 1.5 1.5 0 0 1-3 0Zm-23.23-3.14a1 1 0 0 1-.13 1.4l-2.08 1.74 2.08 1.73a1 1 0 1 1-1.28 1.54l-2.42-2.02a1.63 1.63 0 0 1 0-2.5l2.42-2.02a1 1 0 0 1 1.4.13Zm7.59 1.41a1 1 0 1 1 1.28-1.54l2.42 2.02c.78.65.78 1.85 0 2.5l-2.42 2.02a1 1 0 1 1-1.28-1.54l2.08-1.73-2.08-1.73ZM24.12 18a1 1 0 0 1 .87 1.12l-1 8a1 1 0 1 1-1.98-.24l1-8a1 1 0 0 1 1.11-.87Zm-11.87-5C11.01 13 10 14 10 15.25v15.5c0 1.24 1 2.25 2.25 2.25h17.33c.06 0 .11.02.15.05l4.07 3.05a2 2 0 0 0 3.2-1.6V15.25c0-1.24-1-2.25-2.25-2.25h-22.5ZM12 15.25c0-.14.11-.25.25-.25h22.5c.14 0 .25.11.25.25V34.5l-4.07-3.05a2.2 2.2 0 0 0-1.35-.45H12.25a.25.25 0 0 1-.25-.25v-15.5Zm7.24-10.68a1 1 0 1 0-.48-1.94A22.04 22.04 0 0 0 2.91 17.7a1 1 0 1 0 1.92.58 20.04 20.04 0 0 1 14.4-13.72Zm11.05-1.66a1 1 0 0 0-.58 1.92c6.45 1.92 11.54 7 13.46 13.46a1 1 0 1 0 1.92-.58 22.05 22.05 0 0 0-14.8-14.8ZM4.57 28.76a1 1 0 0 0-1.94.48 22.03 22.03 0 0 0 16.13 16.13 1 1 0 1 0 .48-1.94A20.03 20.03 0 0 1 4.57 28.76Zm40.8.48a1 1 0 1 0-1.94-.48 20.04 20.04 0 0 1-13.72 14.41 1 1 0 0 0 .58 1.92 22.04 22.04 0 0 0 15.08-15.85Z"/></svg>
<h5 class="pt4 fw-bold">Collectives™ on Stack Overflow</h5>
<p class="my16 fs-caption fc-black-500">Find centralized, trusted content and collaborate around the technologies you use most.</p>
<a href="/collectives"
class="js-gps-track s-btn s-btn__filled s-btn__xs"
data-gps-track="top_nav.click({is_current:false, location:2, destination:18})">
Learn more about Collectives
</a>
</div>
</div>
<div class="s-popover ws2"
id="popover-teams-create-cta"
role="menu"
aria-hidden="true">
<div class="s-popover--arrow"></div>
<div class="ps-relative overflow-hidden">
<p class="mb2"><strong>Teams</strong></p>
<p class="mb12 fs-caption fc-black-400">Q&amp;A for work</p>
<p class="mb12 fs-caption fc-black-500">Connect and share knowledge within a single location that is structured and easy to search.</p>
<a href="https://stackoverflow.co/teams/"
class="js-gps-track s-btn s-btn__filled s-btn__xs"
data-gps-track="teams.create.left-sidenav.click({ Action: CtaClick })"
data-ga="[&quot;teams left navigation - anonymous&quot;,&quot;left nav cta&quot;,&quot;stackoverflow.com/teams&quot;,null,null]">
Learn more about Teams
</a>
</div>
<div class="ps-absolute t8 r8">
<svg aria-hidden="true" class="fc-orange-400 svg-spot spotPeople" width="48" height="48" viewBox="0 0 48 48"><path d="M13.5 28a4.5 4.5 0 1 0 0-9 4.5 4.5 0 0 0 0 9ZM7 30a1 1 0 0 1 1-1h11a1 1 0 0 1 1 1v5h11v-5a1 1 0 0 1 1-1h12a1 1 0 0 1 1 1v10a2 2 0 0 1-2 2H33v5a1 1 0 0 1-1 1H20a1 1 0 0 1-1-1v-5H8a1 1 0 0 1-1-1V30Zm25-6.5a4.5 4.5 0 1 0 9 0 4.5 4.5 0 0 0-9 0ZM24.5 34a4.5 4.5 0 1 0 0-9 4.5 4.5 0 0 0 0 9Z" opacity=".2"/><path d="M16.4 26.08A6 6 0 1 0 7.53 26C5.64 26.06 4 27.52 4 29.45V40a1 1 0 0 0 1 1h9a1 1 0 1 0 0-2h-4v-7a1 1 0 1 0-2 0v7H6v-9.55c0-.73.67-1.45 1.64-1.45H16a1 1 0 0 0 .4-1.92ZM12 18a4 4 0 1 1 0 8 4 4 0 0 1 0-8Zm16.47 14a6 6 0 1 0-8.94 0A3.6 3.6 0 0 0 16 35.5V46a1 1 0 0 0 1 1h14a1 1 0 0 0 1-1V35.5c0-1.94-1.64-3.42-3.53-3.5ZM20 28a4 4 0 1 1 8 0 4 4 0 0 1-8 0Zm-.3 6h8.6c1 0 1.7.75 1.7 1.5V45h-2v-7a1 1 0 1 0-2 0v7h-4v-7a1 1 0 1 0-2 0v7h-2v-9.5c0-.75.7-1.5 1.7-1.5ZM42 22c0 1.54-.58 2.94-1.53 4A3.5 3.5 0 0 1 44 29.45V40a1 1 0 0 1-1 1h-9a1 1 0 1 1 0-2h4v-7a1 1 0 1 1 2 0v7h2v-9.55A1.5 1.5 0 0 0 40.48 28H32a1 1 0 0 1-.4-1.92A6 6 0 1 1 42 22Zm-2 0a4 4 0 1 0-8 0 4 4 0 0 0 8 0Z"/><g opacity=".35"><path d="M17 10a1 1 0 011-1h12a1 1 0 110 2H18a1 1 0 01-1-1Zm1-5a1 1 0 100 2h12a1 1 0 100-2H18ZM14 1a1 1 0 00-1 1v12a1 1 0 001 1h5.09l4.2 4.2a1 1 0 001.46-.04l3.7-4.16H34a1 1 0 001-1V2a1 1 0 00-1-1H14Zm1 12V3h18v10h-5a1 1 0 00-.75.34l-3.3 3.7-3.74-3.75a1 1 0 00-.71-.29H15Z"/></g></svg>
</div>
</div>
<div class="s-popover ws2"
id="popover-labs-left-nav"
role="menu"
aria-hidden="true">
<div class="s-popover--arrow"></div>
<svg aria-hidden="true" class="fc-black-600 mb8 svg-icon iconLabs" width="42" height="18"><path d="M11.5 13.624a.374.374 0 0 1-.37.376H5.361a.374.374 0 0 1-.37-.376V4.376c0-.207.165-.376.37-.376H6.62c.204 0 .37.169.37.376v7.611h4.138c.205 0 .371.169.371.377v1.26zm9.432.215c-.07.1-.185.161-.308.161H19.13a.376.376 0 0 1-.356-.254l-.55-1.7h-3.111l-.55 1.7a.377.377 0 0 1-.355.254h-1.494a.376.376 0 0 1-.353-.506l3.39-9.247A.376.376 0 0 1 16.103 4h1.13c.158 0 .299.099.353.247l3.39 9.247a.376.376 0 0 1-.045.345zm-4.157-7.386l-1.219 3.531h2.266l-1.047-3.53zm13.335 5.71a.37.37 0 0 0-.003.524c.956.971 2.047 1.313 3.486 1.313 1.014 0 1.92-.265 2.582-.788.67-.53 1.063-1.306 1.063-2.255 0-.855-.268-1.622-.867-2.145-.456-.41-1.008-.633-1.89-.767l-1.037-.153c-.377-.057-.672-.19-.832-.332-.146-.132-.221-.315-.221-.568 0-.309.11-.56.306-.737.199-.179.518-.312.986-.312.708 0 1.254.151 1.726.601a.37.37 0 0 0 .516-.004l.883-.87a.37.37 0 0 0-.008-.534C35.942 4.334 35.004 4 33.721 4c-1.016 0-1.872.292-2.479.836-.61.548-.935 1.32-.935 2.207 0 .82.243 1.502.781 2.01h.001c.468.437 1.135.716 1.93.826l1.072.153c.508.073.647.147.795.286l.008.007c.14.125.234.34.234.67 0 .332-.124.567-.344.73-.235.174-.617.293-1.165.293-.867 0-1.49-.185-2.066-.76a.37.37 0 0 0-.522-.003l-.92.908zM22.37 14a.374.374 0 0 1-.37-.376V4.376c0-.207.166-.376.37-.376h3.543c.913 0 1.697.264 2.257.78.564.519.863 1.259.863 2.129 0 .845-.377 1.524-.87 1.947.57.433 1.01 1.145 1.01 2.157 0 .941-.317 1.702-.894 2.224-.57.517-1.354.763-2.225.763H22.37zm3.543-1.977c.96 0 .959-1.01.959-1.01s0-1.013-.959-1.013H24v2.023h1.913zm-.115-4.063c1.074 0 1.074-1.015 1.074-1.015s0-1.016-1.074-1.016H24V7.96h1.798z" fill="var(--black-600)"/><path d="M0 4v10a4 4 0 0 0 4 4h34a4 4 0 0 0 4-4V4a4 4 0 0 0-4-4H4a4 4 0 0 0-4 4zm4-2h34a2 2 0 0 1 2 2v10a2 2 0 0 1-2 2H4a2 2 0 0 1-2-2V4a2 2 0 0 1 2-2z" fill="var(--black-600)"/></svg>
<p class="fs-caption">Get early access and see previews of new features.</p>
<a class="s-btn s-btn__filled s-btn__xs s-btn__icon fs-fine" href="https://stackoverflow.co/labs/"><svg aria-hidden="true" class="svg-icon iconShareSm" width="14" height="14" viewBox="0 0 14 14"><path d="M5 1H3a2 2 0 0 0-2 2v8c0 1.1.9 2 2 2h8a2 2 0 0 0 2-2V9h-2v2H3V3h2V1Zm2 0h6v6h-2V4.5L6.5 9 5 7.5 9.5 3H7V1Z"/></svg> Learn more about Labs</a>
</div>
</div>
<div id="content" class="snippet-hidden">
<div itemprop="mainEntity" itemscope itemtype="https://schema.org/Question">
<link itemprop="image" href="https://cdn.sstatic.net/Sites/stackoverflow/Img/apple-touch-icon.png?v=c78bd457575a">
<div class="inner-content clearfix">
<div id="question-header" class="d-flex sm:fd-column">
<h1 itemprop="name" class="fs-headline1 ow-break-word mb8 flex--item fl1"><a href="/questions/20626994/how-to-calculate-the-inverse-of-the-normal-cumulative-distribution-function-in-p" class="question-hyperlink">How to calculate the inverse of the normal cumulative distribution function in python?</a></h1>
<div class="ml12 aside-cta flex--item print:d-none sm:ml0 sm:mb12 sm:order-first sm:as-end">
<a href="/questions/ask" class="ws-nowrap s-btn s-btn__filled">
Ask Question
</a>
</div>
</div>
<div class="d-flex fw-wrap pb8 mb16 bb bc-black-200">
<div class="flex--item ws-nowrap mr16 mb8" title="2013-12-17 05:57:49Z">
<span class="fc-black-400 mr2">Asked</span>
<time itemprop="dateCreated" datetime="2013-12-17T05:57:49">9 years, 11 months ago</time>
</div>
<div class="flex--item ws-nowrap mr16 mb8">
<span class="fc-black-400 mr2">Modified</span>
<a href="?lastactivity" class="s-link s-link__inherit" title="2022-08-15 14:05:56Z">1 year, 3 months ago</a>
</div>
<div class="flex--item ws-nowrap mb8" title="Viewed 199,067 times">
<span class="fc-black-400 mr2">Viewed</span>
199k times
</div>
</div>
<div id="mainbar" role="main" aria-label="question and answers">
<div class="question js-question" data-questionid="20626994" data-position-on-page="0" data-score="107" id="question">
<style>
</style>
<div class="js-zone-container zone-container-main">
<div id="dfp-tlb" class="everyonelovesstackoverflow everyoneloves__top-leaderboard everyoneloves__leaderboard"></div>
<div class="js-report-ad-button-container " style="width: 728px"></div>
</div>
<div class="post-layout ">
<div class="votecell post-layout--left">
<div class="js-voting-container d-flex jc-center fd-column ai-stretch gs4 fc-black-300" data-post-id="20626994" data-referrer="None">
<button class="js-vote-up-btn flex--item s-btn ba bar-pill c-pointer as-center bc-black-225 fc-black-500 h:bg-theme-primary-200"
data-controller="s-tooltip"
data-s-tooltip-placement="right"
title="This question shows research effort; it is useful and clear"
aria-pressed="false"
aria-label="Up vote"
data-selected-classes="fc-theme-primary bc-theme-primary bg-theme-primary-100"
data-unselected-classes="bc-black-225 fc-black-500 h:bg-theme-primary-200">
<svg aria-hidden="true" class="svg-icon iconArrowUp" width="18" height="18" viewBox="0 0 18 18"><path d="M1 12h16L9 4l-8 8Z"/></svg>
</button>
<div class="js-vote-count flex--item d-flex fd-column ai-center fc-theme-body-font fw-bold fs-subheading py4"
itemprop="upvoteCount"
data-value="107">
107
</div>
<button class="js-vote-down-btn flex--item mb8 s-btn ba bar-pill c-pointer as-center bc-black-225 fc-black-500 h:bg-theme-primary-200"
data-controller="s-tooltip"
data-s-tooltip-placement="right"
title="This question does not show any research effort; it is unclear or not useful"
aria-pressed="false"
aria-label="Down vote"
data-selected-classes="fc-theme-primary bc-theme-primary bg-theme-primary-100"
data-unselected-classes="bc-black-225 fc-black-500 h:bg-theme-primary-200">
<svg aria-hidden="true" class="svg-icon iconArrowDown" width="18" height="18" viewBox="0 0 18 18"><path d="M1 6h16l-8 8-8-8Z"/></svg>
</button>
<button class="js-saves-btn s-btn s-btn__unset c-pointer py4"
type="button"
id="saves-btn-20626994"
data-controller="s-tooltip"
data-s-tooltip-placement="right"
data-s-popover-placement=""
title="Save this question."
aria-pressed="false"
data-post-id="20626994"
data-post-type-id="1"
data-user-privilege-for-post-click="0"
aria-controls=""
data-s-popover-auto-show="false"
>
<svg aria-hidden="true" class="fc-theme-primary-400 js-saves-btn-selected d-none svg-icon iconBookmark" width="18" height="18" viewBox="0 0 18 18"><path d="M3 17V3c0-1.1.9-2 2-2h8a2 2 0 0 1 2 2v14l-6-4-6 4Z"/></svg>
<svg aria-hidden="true" class="js-saves-btn-unselected svg-icon iconBookmarkAlt" width="18" height="18" viewBox="0 0 18 18"><path d="m9 10.6 4 2.66V3H5v10.26l4-2.66ZM3 17V3c0-1.1.9-2 2-2h8a2 2 0 0 1 2 2v14l-6-4-6 4Z"/></svg>
</button>
<a class="js-post-issue flex--item s-btn s-btn__unset c-pointer py6 mx-auto" href="/posts/20626994/timeline" data-shortcut="T" data-ks-title="timeline" data-controller="s-tooltip" data-s-tooltip-placement="right" title="Show activity on this post." aria-label="Timeline"><svg aria-hidden="true" class="mln2 mr0 svg-icon iconHistory" width="19" height="18" viewBox="0 0 19 18"><path d="M3 9a8 8 0 1 1 3.73 6.77L8.2 14.3A6 6 0 1 0 5 9l3.01-.01-4 4-4-4h3L3 9Zm7-4h1.01L11 9.36l3.22 2.1-.6.93L10 10V5Z"/></svg></a>
</div>
</div>
<div class="postcell post-layout--right">
<div class="s-prose js-post-body" itemprop="text">
<p>How do I calculate the inverse of the cumulative distribution function (CDF) of the normal distribution in Python?</p>
<p>Which library should I use? Possibly scipy? </p>
</div>
<div class="mt24 mb12">
<div class="post-taglist d-flex gs4 gsy fd-column">
<div class="d-flex ps-relative fw-wrap">
<ul class='ml0 list-ls-none js-post-tag-list-wrapper d-inline'><li class='d-inline mr4 js-post-tag-list-item'><a href="/questions/tagged/python" class="post-tag" title="show questions tagged &#39;python&#39;" aria-label="show questions tagged &#39;python&#39;" rel="tag" aria-labelledby="tag-python-tooltip-container">python</a></li><li class='d-inline mr4 js-post-tag-list-item'><a href="/questions/tagged/scipy" class="post-tag" title="show questions tagged &#39;scipy&#39;" aria-label="show questions tagged &#39;scipy&#39;" rel="tag" aria-labelledby="tag-scipy-tooltip-container">scipy</a></li><li class='d-inline mr4 js-post-tag-list-item'><a href="/questions/tagged/normal-distribution" class="post-tag" title="show questions tagged &#39;normal-distribution&#39;" aria-label="show questions tagged &#39;normal-distribution&#39;" rel="tag" aria-labelledby="tag-normal-distribution-tooltip-container">normal-distribution</a></li></ul>
</div>
</div>
</div>
<div class="mb0 ">
<div class="mt16 d-flex gs8 gsy fw-wrap jc-end ai-start pt4 mb16">
<div class="flex--item mr16 fl1 w96">
<div class="js-post-menu pt2" data-post-id="20626994" data-post-type-id="1">
<div class="d-flex gs8 s-anchors s-anchors__muted fw-wrap">
<div class="flex--item">
<a href="/q/20626994"
rel="nofollow"
itemprop="url"
class="js-share-link js-gps-track"
title="Short permalink to this question"
data-gps-track="post.click({ item: 2, priv: 0, post_type: 1 })"
data-controller="se-share-sheet"
data-se-share-sheet-title="Share a link to this question"
data-se-share-sheet-subtitle=""
data-se-share-sheet-post-type="question"
data-se-share-sheet-social="facebook twitter devto"
data-se-share-sheet-location="1"
data-se-share-sheet-license-url="https%3a%2f%2fcreativecommons.org%2flicenses%2fby-sa%2f3.0%2f"
data-se-share-sheet-license-name="CC BY-SA 3.0"
data-s-popover-placement="bottom-start">Share</a>
</div>
<div class="flex--item">
<button type="button"
id="btnFollowPost-20626994" class="s-btn s-btn__link js-follow-post js-follow-question js-gps-track"
data-gps-track="post.click({ item: 14, priv: 0, post_type: 1 })"
data-controller="s-tooltip " data-s-tooltip-placement="bottom"
data-s-popover-placement="bottom" aria-controls=""
title="Follow this question to receive notifications">
Follow
</button>
</div>
</div>
<div class="js-menu-popup-container"></div>
</div>
</div>
<div class="post-signature flex--item">
<div class="user-info user-hover">
<div class="user-action-time">
<a href="/posts/20626994/revisions" title="show all edits to this post"
class="js-gps-track"
data-gps-track="post.click({ item: 4, priv: 0, post_type: 1 })">edited <span title='2015-03-31 14:52:45Z' class='relativetime'>Mar 31, 2015 at 14:52</span></a>
</div>
<div class="user-gravatar32">
<a href="/users/3139711/knedlsepp"><div class="gravatar-wrapper-32"><img src="https://i.stack.imgur.com/MemQ8.jpg?s=64&amp;g=1" alt="knedlsepp&#39;s user avatar" width="32" height="32" class="bar-sm"></div></a>
</div>
<div class="user-details">
<a href="/users/3139711/knedlsepp">knedlsepp</a>
<div class="-flair">
<span class="reputation-score" title="reputation score " dir="ltr">6,065</span><span title="3 gold badges" aria-hidden="true"><span class="badge1"></span><span class="badgecount">3</span></span><span class="v-visible-sr">3 gold badges</span><span title="20 silver badges" aria-hidden="true"><span class="badge2"></span><span class="badgecount">20</span></span><span class="v-visible-sr">20 silver badges</span><span title="41 bronze badges" aria-hidden="true"><span class="badge3"></span><span class="badgecount">41</span></span><span class="v-visible-sr">41 bronze badges</span>
</div>
</div>
</div>
</div>
<div class="post-signature owner flex--item">
<div class="user-info ">
<div class="user-action-time">
asked <span title='2013-12-17 05:57:49Z' class='relativetime'>Dec 17, 2013 at 5:57</span>
</div>
<div class="user-gravatar32">
<a href="/users/1624752/yueyoum"><div class="gravatar-wrapper-32"><img src="https://www.gravatar.com/avatar/deabf064baf56e064e1d71fabb49b676?s=64&amp;d=identicon&amp;r=PG" alt="Yueyoum&#39;s user avatar" width="32" height="32" class="bar-sm"></div></a>
</div>
<div class="user-details" itemprop="author" itemscope itemtype="http://schema.org/Person">
<a href="/users/1624752/yueyoum">Yueyoum</a><span class="d-none" itemprop="name">Yueyoum</span>
<div class="-flair">
<span class="reputation-score" title="reputation score " dir="ltr">2,853</span><span title="5 gold badges" aria-hidden="true"><span class="badge1"></span><span class="badgecount">5</span></span><span class="v-visible-sr">5 gold badges</span><span title="23 silver badges" aria-hidden="true"><span class="badge2"></span><span class="badgecount">23</span></span><span class="v-visible-sr">23 silver badges</span><span title="26 bronze badges" aria-hidden="true"><span class="badge3"></span><span class="badgecount">26</span></span><span class="v-visible-sr">26 bronze badges</span>
</div>
</div>
</div>
</div>
</div>
</div>
</div>
<span class="d-none" itemprop="commentCount">3</span>
<div class="post-layout--right js-post-comments-component">
<div id="comments-20626994" class="comments js-comments-container bt bc-black-200 mt12 " data-post-id="20626994" data-min-length="15">
<ul class="comments-list js-comments-list"
data-remaining-comments-count="0"
data-canpost="false"
data-cansee="true"
data-comments-unavailable="false"
data-addlink-disabled="true">
<li id="comment-30873309" class="comment js-comment " data-comment-id="30873309" data-comment-owner-id="1217358" data-comment-score="1">
<div class="js-comment-actions comment-actions">
<div class="comment-score js-comment-score js-comment-edit-hide">
<span title="number of &#x27;useful comment&#x27; votes received"
class="cool">1</span>
</div>
</div>
<div class="comment-text js-comment-text-and-form">
<div class="comment-body js-comment-edit-hide">
<span class="comment-copy">Do you mean the inverse Gaussian distribution (<a href="http://en.wikipedia.org/wiki/Inverse_Gaussian_distribution" rel="nofollow noreferrer">en.wikipedia.org/wiki/Inverse_Gaussian_distribution</a>), or the inverse of the cumulative distribution function of the normal distribution (<a href="http://en.wikipedia.org/wiki/Normal_distribution" rel="nofollow noreferrer">en.wikipedia.org/wiki/Normal_distribution</a>), or something else?</span>
<div class="d-inline-flex ai-center">
&ndash;&nbsp;<a href="/users/1217358/warren-weckesser"
title="111,515 reputation"
class="comment-user">Warren Weckesser</a>
</div>
<span class="comment-date" dir="ltr"><span title='2013-12-17 06:30:48Z, License: CC BY-SA 3.0' class='relativetime-clean'>Dec 17, 2013 at 6:30</span></span>
<span title="this comment was edited 1 time">
<svg aria-hidden="true" class="va-text-bottom o50 svg-icon iconPencilSm" width="14" height="14" viewBox="0 0 14 14"><path fill="#F1B600" d="m2 10.12 6.37-6.43 1.88 1.88L3.88 12H2v-1.88Z"/><path fill="#E87C87" d="m11.1 1.71 1.13 1.12c.2.2.2.51 0 .71L11.1 4.7 9.21 2.86l1.17-1.15c.2-.2.51-.2.71 0Z"/></svg>
</span>
</div>
</div>
</li>
<li id="comment-30873351" class="comment js-comment " data-comment-id="30873351" data-comment-owner-id="1624752" data-comment-score="0">
<div class="js-comment-actions comment-actions">
<div class="comment-score js-comment-score js-comment-edit-hide">
</div>
</div>
<div class="comment-text js-comment-text-and-form">
<div class="comment-body js-comment-edit-hide">
<span class="comment-copy">@WarrenWeckesser the second one: inverse of the cumulative distribution function of the normal distribution</span>
<div class="d-inline-flex ai-center">
&ndash;&nbsp;<a href="/users/1624752/yueyoum"
title="2,853 reputation"
class="comment-user owner">Yueyoum</a>
</div>
<span class="comment-date" dir="ltr"><span title='2013-12-17 06:32:40Z, License: CC BY-SA 3.0' class='relativetime-clean'>Dec 17, 2013 at 6:32</span></span>
<span title="this comment was edited 1 time">
<svg aria-hidden="true" class="va-text-bottom o50 svg-icon iconPencilSm" width="14" height="14" viewBox="0 0 14 14"><path fill="#F1B600" d="m2 10.12 6.37-6.43 1.88 1.88L3.88 12H2v-1.88Z"/><path fill="#E87C87" d="m11.1 1.71 1.13 1.12c.2.2.2.51 0 .71L11.1 4.7 9.21 2.86l1.17-1.15c.2-.2.51-.2.71 0Z"/></svg>
</span>
</div>
</div>
</li>
<li id="comment-30873496" class="comment js-comment " data-comment-id="30873496" data-comment-owner-id="1624752" data-comment-score="1">
<div class="js-comment-actions comment-actions">
<div class="comment-score js-comment-score js-comment-edit-hide">
<span title="number of &#x27;useful comment&#x27; votes received"
class="cool">1</span>
</div>
</div>
<div class="comment-text js-comment-text-and-form">
<div class="comment-body js-comment-edit-hide">
<span class="comment-copy">@WarrenWeckesser i mean the python version of &quot;normsinv&quot; function in excel.</span>
<div class="d-inline-flex ai-center">
&ndash;&nbsp;<a href="/users/1624752/yueyoum"
title="2,853 reputation"
class="comment-user owner">Yueyoum</a>
</div>
<span class="comment-date" dir="ltr"><span title='2013-12-17 06:39:22Z, License: CC BY-SA 3.0' class='relativetime-clean'>Dec 17, 2013 at 6:39</span></span>
</div>
</div>
</li>
</ul>
</div>
<div id="comments-link-20626994" data-rep=50 data-anon=true>
<a class="js-add-link comments-link disabled-link" title="Use comments to ask for more information or suggest improvements. Avoid answering questions in comments." href="#" role="button">Add a comment</a>
<span class="js-link-separator dno">&nbsp;|&nbsp;</span>
<a class="js-show-link comments-link dno" title="Expand to show all comments on this post" href=# onclick="" role="button"></a>
</div>
</div>
</div>
</div>
<div class="js-zone-container zone-container-responsive">
<div id="dfp-isb" class="everyonelovesstackoverflow everyoneloves__inline-sidebar mx-auto"></div>
<div class="js-report-ad-button-container mx-auto" style="width: 300px"></div>
</div>
<div id="answers">
<a name="tab-top"></a>
<div id="answers-header">
<div class="answers-subheader d-flex ai-center mb8">
<div class="flex--item fl1">
<h2 class="mb0" data-answercount="3">
3 Answers
<span style="display:none;" itemprop="answerCount">3</span>
</h2>
</div>
<div class="flex--item">
<div class="d-flex g4 gsx ai-center sm:fd-column sm:ai-start">
<div class="d-flex fd-column ai-end sm:ai-start">
<label class="flex--item fs-caption" for="answer-sort-dropdown-select-menu">
Sorted by:
</label>
<a
class="js-sort-preference-change s-link flex--item fs-fine d-none"
data-value="ScoreDesc"
href="/questions/20626994/how-to-calculate-the-inverse-of-the-normal-cumulative-distribution-function-in-p?answertab=scoredesc#tab-top"
>
Reset to default
</a>
</div>
<div class="flex--item s-select">
<select id="answer-sort-dropdown-select-menu">
<option
value=scoredesc
selected=selected
>
Highest score (default)
</option>
<option
value=trending
>
Trending (recent votes count more)
</option>
<option
value=modifieddesc
>
Date modified (newest first)
</option>
<option
value=createdasc
>
Date created (oldest first)
</option>
</select>
</div>
</div>
</div>
</div>
</div>
<a name="20627638"></a>
<div id="answer-20627638" class="answer js-answer accepted-answer js-accepted-answer" data-answerid="20627638" data-parentid="20626994" data-score="171" data-position-on-page="1" data-highest-scored="1" data-question-has-accepted-highest-score="1" itemprop="acceptedAnswer" itemscope itemtype="https://schema.org/Answer">
<div class="post-layout">
<div class="votecell post-layout--left">
<div class="js-voting-container d-flex jc-center fd-column ai-stretch gs4 fc-black-300" data-post-id="20627638" data-referrer="None">
<button class="js-vote-up-btn flex--item s-btn ba bar-pill c-pointer as-center bc-black-225 fc-black-500 h:bg-theme-primary-200"
data-controller="s-tooltip"
data-s-tooltip-placement="right"
title="This answer is useful"
aria-pressed="false"
aria-label="Up vote"
data-selected-classes="fc-theme-primary bc-theme-primary bg-theme-primary-100"
data-unselected-classes="bc-black-225 fc-black-500 h:bg-theme-primary-200">
<svg aria-hidden="true" class="svg-icon iconArrowUp" width="18" height="18" viewBox="0 0 18 18"><path d="M1 12h16L9 4l-8 8Z"/></svg>
</button>
<div class="js-vote-count flex--item d-flex fd-column ai-center fc-theme-body-font fw-bold fs-subheading py4"
itemprop="upvoteCount"
data-value="171">
171
</div>
<button class="js-vote-down-btn flex--item mb8 s-btn ba bar-pill c-pointer as-center bc-black-225 fc-black-500 h:bg-theme-primary-200"
data-controller="s-tooltip"
data-s-tooltip-placement="right"
title="This answer is not useful"
aria-pressed="false"
aria-label="Down vote"
data-selected-classes="fc-theme-primary bc-theme-primary bg-theme-primary-100"
data-unselected-classes="bc-black-225 fc-black-500 h:bg-theme-primary-200">
<svg aria-hidden="true" class="svg-icon iconArrowDown" width="18" height="18" viewBox="0 0 18 18"><path d="M1 6h16l-8 8-8-8Z"/></svg>
</button>
<button class="js-saves-btn s-btn s-btn__unset c-pointer py4"
type="button"
id="saves-btn-20627638"
data-controller="s-tooltip"
data-s-tooltip-placement="right"
data-s-popover-placement=""
title="Save this answer."
aria-pressed="false"
data-post-id="20627638"
data-post-type-id="2"
data-user-privilege-for-post-click="0"
aria-controls=""
data-s-popover-auto-show="false"
>
<svg aria-hidden="true" class="fc-theme-primary-400 js-saves-btn-selected d-none svg-icon iconBookmark" width="18" height="18" viewBox="0 0 18 18"><path d="M3 17V3c0-1.1.9-2 2-2h8a2 2 0 0 1 2 2v14l-6-4-6 4Z"/></svg>
<svg aria-hidden="true" class="js-saves-btn-unselected svg-icon iconBookmarkAlt" width="18" height="18" viewBox="0 0 18 18"><path d="m9 10.6 4 2.66V3H5v10.26l4-2.66ZM3 17V3c0-1.1.9-2 2-2h8a2 2 0 0 1 2 2v14l-6-4-6 4Z"/></svg>
</button>
<div class="js-accepted-answer-indicator flex--item fc-green-400 py6 mtn8" data-s-tooltip-placement="right" title="Loading when this answer was accepted&#x2026;" tabindex="0" role="note" aria-label="Accepted">
<div class="ta-center">
<svg aria-hidden="true" class="svg-icon iconCheckmarkLg" width="36" height="36" viewBox="0 0 36 36"><path d="m6 14 8 8L30 6v8L14 30l-8-8v-8Z"/></svg>
</div>
</div>
<a class="js-post-issue flex--item s-btn s-btn__unset c-pointer py6 mx-auto" href="/posts/20627638/timeline" data-shortcut="T" data-ks-title="timeline" data-controller="s-tooltip" data-s-tooltip-placement="right" title="Show activity on this post." aria-label="Timeline"><svg aria-hidden="true" class="mln2 mr0 svg-icon iconHistory" width="19" height="18" viewBox="0 0 19 18"><path d="M3 9a8 8 0 1 1 3.73 6.77L8.2 14.3A6 6 0 1 0 5 9l3.01-.01-4 4-4-4h3L3 9Zm7-4h1.01L11 9.36l3.22 2.1-.6.93L10 10V5Z"/></svg></a>
</div>
</div>
<div class="answercell post-layout--right">
<div class="s-prose js-post-body" itemprop="text">
<p><a href="http://support.microsoft.com/kb/826772" rel="noreferrer">NORMSINV</a> (mentioned in a comment) is the inverse of the CDF of the standard normal distribution. Using <code>scipy</code>, you can compute this with the <code>ppf</code> method of the <a href="http://docs.scipy.org/doc/scipy/reference/generated/scipy.stats.norm.html" rel="noreferrer"><code>scipy.stats.norm</code></a> object. The acronym <code>ppf</code> stands for <a href="http://www.itl.nist.gov/div898/handbook/eda/section3/eda362.htm#PPF" rel="noreferrer"><em>percent point function</em></a>, which is another name for the <a href="https://en.wikipedia.org/wiki/Quantile_function" rel="noreferrer"><em>quantile function</em></a>.</p>
<pre><code>In [20]: from scipy.stats import norm
In [21]: norm.ppf(0.95)
Out[21]: 1.6448536269514722
</code></pre>
<p>Check that it is the inverse of the CDF:</p>
<pre><code>In [34]: norm.cdf(norm.ppf(0.95))
Out[34]: 0.94999999999999996
</code></pre>
<p>By default, <code>norm.ppf</code> uses mean=0 and stddev=1, which is the &quot;standard&quot; normal distribution. You can use a different mean and standard deviation by specifying the <code>loc</code> and <code>scale</code> arguments, respectively.</p>
<pre><code>In [35]: norm.ppf(0.95, loc=10, scale=2)
Out[35]: 13.289707253902945
</code></pre>
<p>If you look at the source code for <code>scipy.stats.norm</code>, you'll find that the <code>ppf</code> method ultimately calls <a href="http://docs.scipy.org/doc/scipy/reference/generated/scipy.special.ndtri.html" rel="noreferrer"><code>scipy.special.ndtri</code></a>. So to compute the inverse of the CDF of the standard normal distribution, you could use that function directly:</p>
<pre><code>In [43]: from scipy.special import ndtri
In [44]: ndtri(0.95)
Out[44]: 1.6448536269514722
</code></pre>
<p><code>ndtri</code> is <em>much</em> faster than <code>norm.ppf</code>:</p>
<pre><code>In [46]: %timeit norm.ppf(0.95)
240 µs ± 1.75 µs per loop (mean ± std. dev. of 7 runs, 1,000 loops each)
In [47]: %timeit ndtri(0.95)
1.47 µs ± 1.3 ns per loop (mean ± std. dev. of 7 runs, 1,000,000 loops each)
</code></pre>
</div>
<div class="mt24">
<div class="d-flex fw-wrap ai-start jc-end gs8 gsy">
<time itemprop="dateCreated" datetime="2013-12-17T06:44:02"></time>
<div class="flex--item mr16" style="flex: 1 1 100px;">
<div class="js-post-menu pt2" data-post-id="20627638" data-post-type-id="2">
<div class="d-flex gs8 s-anchors s-anchors__muted fw-wrap">
<div class="flex--item">
<a href="/a/20627638"
rel="nofollow"
itemprop="url"
class="js-share-link js-gps-track"
title="Short permalink to this answer"
data-gps-track="post.click({ item: 2, priv: 0, post_type: 2 })"
data-controller="se-share-sheet"
data-se-share-sheet-title="Share a link to this answer"
data-se-share-sheet-subtitle=""
data-se-share-sheet-post-type="answer"
data-se-share-sheet-social="facebook twitter devto"
data-se-share-sheet-location="2"
data-se-share-sheet-license-url="https%3a%2f%2fcreativecommons.org%2flicenses%2fby-sa%2f4.0%2f"
data-se-share-sheet-license-name="CC BY-SA 4.0"
data-s-popover-placement="bottom-start">Share</a>
</div>
<div class="flex--item">
<button type="button"
id="btnFollowPost-20627638" class="s-btn s-btn__link js-follow-post js-follow-answer js-gps-track"
data-gps-track="post.click({ item: 14, priv: 0, post_type: 2 })"
data-controller="s-tooltip " data-s-tooltip-placement="bottom"
data-s-popover-placement="bottom" aria-controls=""
title="Follow this answer to receive notifications">
Follow
</button>
</div>
</div>
<div class="js-menu-popup-container"></div>
</div>
</div>
<div class="post-signature flex--item fl0">
<div class="user-info ">
<div class="user-action-time">
<a href="/posts/20627638/revisions" title="show all edits to this post"
class="js-gps-track"
data-gps-track="post.click({ item: 4, priv: 0, post_type: 2 })">edited <span title='2022-08-15 14:05:56Z' class='relativetime'>Aug 15, 2022 at 14:05</span></a>
</div>
<div class="user-gravatar32">
</div>
<div class="user-details">
<div class="-flair">
</div>
</div>
</div>
</div>
<div class="post-signature flex--item fl0">
<div class="user-info ">
<div class="user-action-time">
answered <span title='2013-12-17 06:44:02Z' class='relativetime'>Dec 17, 2013 at 6:44</span>
</div>
<div class="user-gravatar32">
<a href="/users/1217358/warren-weckesser"><div class="gravatar-wrapper-32"><img src="https://www.gravatar.com/avatar/d2aafb97833979e3668c61d36e697bfc?s=64&amp;d=identicon&amp;r=PG" alt="Warren Weckesser&#39;s user avatar" width="32" height="32" class="bar-sm"></div></a>
</div>
<div class="user-details" itemprop="author" itemscope itemtype="http://schema.org/Person">
<a href="/users/1217358/warren-weckesser">Warren Weckesser</a><span class="d-none" itemprop="name">Warren Weckesser</span>
<div class="-flair">
<span class="reputation-score" title="reputation score 111,515" dir="ltr">112k</span><span title="19 gold badges" aria-hidden="true"><span class="badge1"></span><span class="badgecount">19</span></span><span class="v-visible-sr">19 gold badges</span><span title="196 silver badges" aria-hidden="true"><span class="badge2"></span><span class="badgecount">196</span></span><span class="v-visible-sr">196 silver badges</span><span title="215 bronze badges" aria-hidden="true"><span class="badge3"></span><span class="badgecount">215</span></span><span class="v-visible-sr">215 bronze badges</span>
</div>
</div>
</div>
</div>
</div>
</div>
</div>
<span class="d-none" itemprop="commentCount">4</span>
<div class="post-layout--right js-post-comments-component">
<div id="comments-20627638" class="comments js-comments-container bt bc-black-200 mt12 " data-post-id="20627638" data-min-length="15">
<ul class="comments-list js-comments-list"
data-remaining-comments-count="0"
data-canpost="false"
data-cansee="true"
data-comments-unavailable="false"
data-addlink-disabled="true">
<li id="comment-41064090" class="comment js-comment " data-comment-id="41064090" data-comment-owner-id="1670078" data-comment-score="29">
<div class="js-comment-actions comment-actions">
<div class="comment-score js-comment-score js-comment-edit-hide">
<span title="number of &#x27;useful comment&#x27; votes received"
class="hot">29</span>
</div>
</div>
<div class="comment-text js-comment-text-and-form">
<div class="comment-body js-comment-edit-hide">
<span class="comment-copy">I always think &quot;percent point function&quot; (ppf) is a terrible name. Most people in statistics just use &quot;quantile function&quot;.</span>
<div class="d-inline-flex ai-center">
&ndash;&nbsp;<a href="/users/1670078/william-zhang"
title="699 reputation"
class="comment-user">William Zhang</a>
</div>
<span class="comment-date" dir="ltr"><span title='2014-10-04 00:44:33Z, License: CC BY-SA 3.0' class='relativetime-clean'>Oct 4, 2014 at 0:44</span></span>
</div>
</div>
</li>
<li id="comment-116623338" class="comment js-comment " data-comment-id="116623338" data-comment-owner-id="1391660" data-comment-score="0">
<div class="js-comment-actions comment-actions">
<div class="comment-score js-comment-score js-comment-edit-hide">
</div>
</div>
<div class="comment-text js-comment-text-and-form">
<div class="comment-body js-comment-edit-hide">
<span class="comment-copy">Don&#39;t you need to specify the mean and the std on both ppf and cdf?</span>
<div class="d-inline-flex ai-center">
&ndash;&nbsp;<a href="/users/1391660/bones-felipe"
title="586 reputation"
class="comment-user">bones.felipe</a>
</div>
<span class="comment-date" dir="ltr"><span title='2021-01-29 19:23:44Z, License: CC BY-SA 4.0' class='relativetime-clean'>Jan 29, 2021 at 19:23</span></span>
<span title="this comment was edited 2 times">
<svg aria-hidden="true" class="va-text-bottom o50 svg-icon iconPencilSm" width="14" height="14" viewBox="0 0 14 14"><path fill="#F1B600" d="m2 10.12 6.37-6.43 1.88 1.88L3.88 12H2v-1.88Z"/><path fill="#E87C87" d="m11.1 1.71 1.13 1.12c.2.2.2.51 0 .71L11.1 4.7 9.21 2.86l1.17-1.15c.2-.2.51-.2.71 0Z"/></svg>
</span>
</div>
</div>
</li>
<li id="comment-116623958" class="comment js-comment " data-comment-id="116623958" data-comment-owner-id="1217358" data-comment-score="0">
<div class="js-comment-actions comment-actions">
<div class="comment-score js-comment-score js-comment-edit-hide">
</div>
</div>
<div class="comment-text js-comment-text-and-form">
<div class="comment-body js-comment-edit-hide">
<span class="comment-copy">@bones.felipe, the &quot;standard&quot; normal distribution has mean 0 and standard deviation 1. These are the default values for the location and scale of the <code>scipy.stats.norm</code> methods.</span>
<div class="d-inline-flex ai-center">
&ndash;&nbsp;<a href="/users/1217358/warren-weckesser"
title="111,515 reputation"
class="comment-user">Warren Weckesser</a>
</div>
<span class="comment-date" dir="ltr"><span title='2021-01-29 19:55:51Z, License: CC BY-SA 4.0' class='relativetime-clean'>Jan 29, 2021 at 19:55</span></span>
</div>
</div>
</li>
<li id="comment-116630954" class="comment js-comment " data-comment-id="116630954" data-comment-owner-id="1391660" data-comment-score="0">
<div class="js-comment-actions comment-actions">
<div class="comment-score js-comment-score js-comment-edit-hide">
</div>
</div>
<div class="comment-text js-comment-text-and-form">
<div class="comment-body js-comment-edit-hide">
<span class="comment-copy">Right, I thought I saw this <code>norm.cdf(norm.ppf(0.95, loc=10, scale=2))</code> and I thought it was weird <code>norm.cdf</code> did not have <code>loc=10</code> and <code>scale=2</code> too, I guess it should.</span>
<div class="d-inline-flex ai-center">
&ndash;&nbsp;<a href="/users/1391660/bones-felipe"
title="586 reputation"
class="comment-user">bones.felipe</a>
</div>
<span class="comment-date" dir="ltr"><span title='2021-01-30 05:33:43Z, License: CC BY-SA 4.0' class='relativetime-clean'>Jan 30, 2021 at 5:33</span></span>
<span title="this comment was edited 1 time">
<svg aria-hidden="true" class="va-text-bottom o50 svg-icon iconPencilSm" width="14" height="14" viewBox="0 0 14 14"><path fill="#F1B600" d="m2 10.12 6.37-6.43 1.88 1.88L3.88 12H2v-1.88Z"/><path fill="#E87C87" d="m11.1 1.71 1.13 1.12c.2.2.2.51 0 .71L11.1 4.7 9.21 2.86l1.17-1.15c.2-.2.51-.2.71 0Z"/></svg>
</span>
</div>
</div>
</li>
</ul>
</div>
<div id="comments-link-20627638" data-rep=50 data-anon=true>
<a class="js-add-link comments-link disabled-link" title="Use comments to ask for more information or suggest improvements. Avoid comments like &#x201C;&#x2B;1&#x201D; or &#x201C;thanks&#x201D;." href="#" role="button">Add a comment</a>
<span class="js-link-separator dno">&nbsp;|&nbsp;</span>
<a class="js-show-link comments-link dno" title="Expand to show all comments on this post" href=# onclick="" role="button"></a>
</div>
</div>
</div>
</div>
<div class="js-zone-container zone-container-main">
<div id="dfp-mlb" class="everyonelovesstackoverflow everyoneloves__mid-leaderboard everyoneloves__leaderboard"></div>
<div class="js-report-ad-button-container " style="width: 728px"></div>
</div>
<a name="55250607"></a>
<div id="answer-55250607" class="answer js-answer" data-answerid="55250607" data-parentid="20626994" data-score="39" data-position-on-page="2" data-highest-scored="0" data-question-has-accepted-highest-score="1" itemprop="suggestedAnswer" itemscope itemtype="https://schema.org/Answer">
<div class="post-layout">
<div class="votecell post-layout--left">
<div class="js-voting-container d-flex jc-center fd-column ai-stretch gs4 fc-black-300" data-post-id="55250607" data-referrer="None">
<button class="js-vote-up-btn flex--item s-btn ba bar-pill c-pointer as-center bc-black-225 fc-black-500 h:bg-theme-primary-200"
data-controller="s-tooltip"
data-s-tooltip-placement="right"
title="This answer is useful"
aria-pressed="false"
aria-label="Up vote"
data-selected-classes="fc-theme-primary bc-theme-primary bg-theme-primary-100"
data-unselected-classes="bc-black-225 fc-black-500 h:bg-theme-primary-200">
<svg aria-hidden="true" class="svg-icon iconArrowUp" width="18" height="18" viewBox="0 0 18 18"><path d="M1 12h16L9 4l-8 8Z"/></svg>
</button>
<div class="js-vote-count flex--item d-flex fd-column ai-center fc-theme-body-font fw-bold fs-subheading py4"
itemprop="upvoteCount"
data-value="39">
39
</div>
<button class="js-vote-down-btn flex--item mb8 s-btn ba bar-pill c-pointer as-center bc-black-225 fc-black-500 h:bg-theme-primary-200"
data-controller="s-tooltip"
data-s-tooltip-placement="right"
title="This answer is not useful"
aria-pressed="false"
aria-label="Down vote"
data-selected-classes="fc-theme-primary bc-theme-primary bg-theme-primary-100"
data-unselected-classes="bc-black-225 fc-black-500 h:bg-theme-primary-200">
<svg aria-hidden="true" class="svg-icon iconArrowDown" width="18" height="18" viewBox="0 0 18 18"><path d="M1 6h16l-8 8-8-8Z"/></svg>
</button>
<button class="js-saves-btn s-btn s-btn__unset c-pointer py4"
type="button"
id="saves-btn-55250607"
data-controller="s-tooltip"
data-s-tooltip-placement="right"
data-s-popover-placement=""
title="Save this answer."
aria-pressed="false"
data-post-id="55250607"
data-post-type-id="2"
data-user-privilege-for-post-click="0"
aria-controls=""
data-s-popover-auto-show="false"
>
<svg aria-hidden="true" class="fc-theme-primary-400 js-saves-btn-selected d-none svg-icon iconBookmark" width="18" height="18" viewBox="0 0 18 18"><path d="M3 17V3c0-1.1.9-2 2-2h8a2 2 0 0 1 2 2v14l-6-4-6 4Z"/></svg>
<svg aria-hidden="true" class="js-saves-btn-unselected svg-icon iconBookmarkAlt" width="18" height="18" viewBox="0 0 18 18"><path d="m9 10.6 4 2.66V3H5v10.26l4-2.66ZM3 17V3c0-1.1.9-2 2-2h8a2 2 0 0 1 2 2v14l-6-4-6 4Z"/></svg>
</button>
<div class="js-accepted-answer-indicator flex--item fc-green-400 py6 mtn8 d-none" data-s-tooltip-placement="right" title="Loading when this answer was accepted&#x2026;" tabindex="0" role="note" aria-label="Accepted">
<div class="ta-center">
<svg aria-hidden="true" class="svg-icon iconCheckmarkLg" width="36" height="36" viewBox="0 0 36 36"><path d="m6 14 8 8L30 6v8L14 30l-8-8v-8Z"/></svg>
</div>
</div>
<a class="js-post-issue flex--item s-btn s-btn__unset c-pointer py6 mx-auto" href="/posts/55250607/timeline" data-shortcut="T" data-ks-title="timeline" data-controller="s-tooltip" data-s-tooltip-placement="right" title="Show activity on this post." aria-label="Timeline"><svg aria-hidden="true" class="mln2 mr0 svg-icon iconHistory" width="19" height="18" viewBox="0 0 19 18"><path d="M3 9a8 8 0 1 1 3.73 6.77L8.2 14.3A6 6 0 1 0 5 9l3.01-.01-4 4-4-4h3L3 9Zm7-4h1.01L11 9.36l3.22 2.1-.6.93L10 10V5Z"/></svg></a>
</div>
</div>
<div class="answercell post-layout--right">
<div class="s-prose js-post-body" itemprop="text">
<p>Starting <code>Python 3.8</code>, the standard library provides the <a href="https://docs.python.org/3.8/library/statistics.html?highlight=normaldist#statistics.NormalDist" rel="noreferrer"><code>NormalDist</code></a> object as part of the <a href="https://docs.python.org/3.8/library/statistics.html" rel="noreferrer"><code>statistics</code></a> module.</p>
<p>It can be used to get the <strong><em>inverse cumulative distribution function</em></strong> (<strong><a href="https://docs.python.org/3.8/library/statistics.html#statistics.NormalDist.inv_cdf" rel="noreferrer"><code>inv_cdf</code></a></strong> - inverse of the <a href="https://docs.python.org/3.8/library/statistics.html#statistics.NormalDist.cdf" rel="noreferrer"><code>cdf</code></a>), also known as the <strong>quantile function</strong> or the <strong>percent-point function</strong> for a given <em>mean</em> (<code>mu</code>) and <em>standard deviation</em> (<code>sigma</code>):</p>
<pre><code>from statistics import NormalDist
NormalDist(mu=10, sigma=2).inv_cdf(0.95)
# 13.289707253902943
</code></pre>
<p>Which can be simplified for the <em>standard normal distribution</em> (<code>mu = 0</code> and <code>sigma = 1</code>):</p>
<pre><code>NormalDist().inv_cdf(0.95)
# 1.6448536269514715
</code></pre>
</div>
<div class="mt24">
<div class="d-flex fw-wrap ai-start jc-end gs8 gsy">
<time itemprop="dateCreated" datetime="2019-03-19T21:58:57"></time>
<div class="flex--item mr16" style="flex: 1 1 100px;">
<div class="js-post-menu pt2" data-post-id="55250607" data-post-type-id="2">
<div class="d-flex gs8 s-anchors s-anchors__muted fw-wrap">
<div class="flex--item">
<a href="/a/55250607"
rel="nofollow"
itemprop="url"
class="js-share-link js-gps-track"
title="Short permalink to this answer"
data-gps-track="post.click({ item: 2, priv: 0, post_type: 2 })"
data-controller="se-share-sheet"
data-se-share-sheet-title="Share a link to this answer"
data-se-share-sheet-subtitle=""
data-se-share-sheet-post-type="answer"
data-se-share-sheet-social="facebook twitter devto"
data-se-share-sheet-location="2"
data-se-share-sheet-license-url="https%3a%2f%2fcreativecommons.org%2flicenses%2fby-sa%2f4.0%2f"
data-se-share-sheet-license-name="CC BY-SA 4.0"
data-s-popover-placement="bottom-start">Share</a>
</div>
<div class="flex--item">
<button type="button"
id="btnFollowPost-55250607" class="s-btn s-btn__link js-follow-post js-follow-answer js-gps-track"
data-gps-track="post.click({ item: 14, priv: 0, post_type: 2 })"
data-controller="s-tooltip " data-s-tooltip-placement="bottom"
data-s-popover-placement="bottom" aria-controls=""
title="Follow this answer to receive notifications">
Follow
</button>
</div>
</div>
<div class="js-menu-popup-container"></div>
</div>
</div>
<div class="post-signature flex--item fl0">
<div class="user-info user-hover">
<div class="user-action-time">
answered <span title='2019-03-19 21:58:57Z' class='relativetime'>Mar 19, 2019 at 21:58</span>
</div>
<div class="user-gravatar32">
<a href="/users/9297144/xavier-guihot"><div class="gravatar-wrapper-32"><img src="https://i.stack.imgur.com/WLMMD.png?s=64&amp;g=1" alt="Xavier Guihot&#39;s user avatar" width="32" height="32" class="bar-sm"></div></a>
</div>
<div class="user-details" itemprop="author" itemscope itemtype="http://schema.org/Person">
<a href="/users/9297144/xavier-guihot">Xavier Guihot</a><span class="d-none" itemprop="name">Xavier Guihot</span>
<div class="-flair">
<span class="reputation-score" title="reputation score 56,556" dir="ltr">56.6k</span><span title="22 gold badges" aria-hidden="true"><span class="badge1"></span><span class="badgecount">22</span></span><span class="v-visible-sr">22 gold badges</span><span title="297 silver badges" aria-hidden="true"><span class="badge2"></span><span class="badgecount">297</span></span><span class="v-visible-sr">297 silver badges</span><span title="192 bronze badges" aria-hidden="true"><span class="badge3"></span><span class="badgecount">192</span></span><span class="v-visible-sr">192 bronze badges</span>
</div>
</div>
</div>
</div>
</div>
</div>
</div>
<span class="d-none" itemprop="commentCount">2</span>
<div class="post-layout--right js-post-comments-component">
<div id="comments-55250607" class="comments js-comments-container bt bc-black-200 mt12 " data-post-id="55250607" data-min-length="15">
<ul class="comments-list js-comments-list"
data-remaining-comments-count="0"
data-canpost="false"
data-cansee="true"
data-comments-unavailable="false"
data-addlink-disabled="true">
<li id="comment-106743999" class="comment js-comment " data-comment-id="106743999" data-comment-owner-id="5437918" data-comment-score="4">
<div class="js-comment-actions comment-actions">
<div class="comment-score js-comment-score js-comment-edit-hide">
<span title="number of &#x27;useful comment&#x27; votes received"
class="cool">4</span>
</div>
</div>
<div class="comment-text js-comment-text-and-form">
<div class="comment-body js-comment-edit-hide">
<span class="comment-copy">Great tip! This allows me to drop the dependency on scipy, which I needed just for the single stats.norm.ppf method</span>
<div class="d-inline-flex ai-center">
&ndash;&nbsp;<a href="/users/5437918/jethro-cao"
title="1,010 reputation"
class="comment-user">Jethro Cao</a>
</div>
<span class="comment-date" dir="ltr"><span title='2020-02-21 16:56:27Z, License: CC BY-SA 4.0' class='relativetime-clean'>Feb 21, 2020 at 16:56</span></span>
</div>
</div>
</li>
<li id="comment-126713418" class="comment js-comment " data-comment-id="126713418" data-comment-owner-id="10824677" data-comment-score="0">
<div class="js-comment-actions comment-actions">
<div class="comment-score js-comment-score js-comment-edit-hide">
</div>
</div>
<div class="comment-text js-comment-text-and-form">
<div class="comment-body js-comment-edit-hide">
<span class="comment-copy">can you use that to transform data with uniform distribution to normal ?</span>
<div class="d-inline-flex ai-center">
&ndash;&nbsp;<a href="/users/10824677/vanetoj"
title="103 reputation"
class="comment-user">vanetoj</a>
</div>
<span class="comment-date" dir="ltr"><span title='2022-03-31 20:51:01Z, License: CC BY-SA 4.0' class='relativetime-clean'>Mar 31, 2022 at 20:51</span></span>
</div>
</div>
</li>
</ul>
</div>
<div id="comments-link-55250607" data-rep=50 data-anon=true>
<a class="js-add-link comments-link disabled-link" title="Use comments to ask for more information or suggest improvements. Avoid comments like &#x201C;&#x2B;1&#x201D; or &#x201C;thanks&#x201D;." href="#" role="button">Add a comment</a>
<span class="js-link-separator dno">&nbsp;|&nbsp;</span>
<a class="js-show-link comments-link dno" title="Expand to show all comments on this post" href=# onclick="" role="button"></a>
</div>
</div>
</div>
</div>
<a name="32798562"></a>
<div id="answer-32798562" class="answer js-answer" data-answerid="32798562" data-parentid="20626994" data-score="21" data-position-on-page="3" data-highest-scored="0" data-question-has-accepted-highest-score="1" itemprop="suggestedAnswer" itemscope itemtype="https://schema.org/Answer">
<div class="post-layout">
<div class="votecell post-layout--left">
<div class="js-voting-container d-flex jc-center fd-column ai-stretch gs4 fc-black-300" data-post-id="32798562" data-referrer="None">
<button class="js-vote-up-btn flex--item s-btn ba bar-pill c-pointer as-center bc-black-225 fc-black-500 h:bg-theme-primary-200"
data-controller="s-tooltip"
data-s-tooltip-placement="right"
title="This answer is useful"
aria-pressed="false"
aria-label="Up vote"
data-selected-classes="fc-theme-primary bc-theme-primary bg-theme-primary-100"
data-unselected-classes="bc-black-225 fc-black-500 h:bg-theme-primary-200">
<svg aria-hidden="true" class="svg-icon iconArrowUp" width="18" height="18" viewBox="0 0 18 18"><path d="M1 12h16L9 4l-8 8Z"/></svg>
</button>
<div class="js-vote-count flex--item d-flex fd-column ai-center fc-theme-body-font fw-bold fs-subheading py4"
itemprop="upvoteCount"
data-value="21">
21
</div>
<button class="js-vote-down-btn flex--item mb8 s-btn ba bar-pill c-pointer as-center bc-black-225 fc-black-500 h:bg-theme-primary-200"
data-controller="s-tooltip"
data-s-tooltip-placement="right"
title="This answer is not useful"
aria-pressed="false"
aria-label="Down vote"
data-selected-classes="fc-theme-primary bc-theme-primary bg-theme-primary-100"
data-unselected-classes="bc-black-225 fc-black-500 h:bg-theme-primary-200">
<svg aria-hidden="true" class="svg-icon iconArrowDown" width="18" height="18" viewBox="0 0 18 18"><path d="M1 6h16l-8 8-8-8Z"/></svg>
</button>
<button class="js-saves-btn s-btn s-btn__unset c-pointer py4"
type="button"
id="saves-btn-32798562"
data-controller="s-tooltip"
data-s-tooltip-placement="right"
data-s-popover-placement=""
title="Save this answer."
aria-pressed="false"
data-post-id="32798562"
data-post-type-id="2"
data-user-privilege-for-post-click="0"
aria-controls=""
data-s-popover-auto-show="false"
>
<svg aria-hidden="true" class="fc-theme-primary-400 js-saves-btn-selected d-none svg-icon iconBookmark" width="18" height="18" viewBox="0 0 18 18"><path d="M3 17V3c0-1.1.9-2 2-2h8a2 2 0 0 1 2 2v14l-6-4-6 4Z"/></svg>
<svg aria-hidden="true" class="js-saves-btn-unselected svg-icon iconBookmarkAlt" width="18" height="18" viewBox="0 0 18 18"><path d="m9 10.6 4 2.66V3H5v10.26l4-2.66ZM3 17V3c0-1.1.9-2 2-2h8a2 2 0 0 1 2 2v14l-6-4-6 4Z"/></svg>
</button>
<div class="js-accepted-answer-indicator flex--item fc-green-400 py6 mtn8 d-none" data-s-tooltip-placement="right" title="Loading when this answer was accepted&#x2026;" tabindex="0" role="note" aria-label="Accepted">
<div class="ta-center">
<svg aria-hidden="true" class="svg-icon iconCheckmarkLg" width="36" height="36" viewBox="0 0 36 36"><path d="m6 14 8 8L30 6v8L14 30l-8-8v-8Z"/></svg>
</div>
</div>
<a class="js-post-issue flex--item s-btn s-btn__unset c-pointer py6 mx-auto" href="/posts/32798562/timeline" data-shortcut="T" data-ks-title="timeline" data-controller="s-tooltip" data-s-tooltip-placement="right" title="Show activity on this post." aria-label="Timeline"><svg aria-hidden="true" class="mln2 mr0 svg-icon iconHistory" width="19" height="18" viewBox="0 0 19 18"><path d="M3 9a8 8 0 1 1 3.73 6.77L8.2 14.3A6 6 0 1 0 5 9l3.01-.01-4 4-4-4h3L3 9Zm7-4h1.01L11 9.36l3.22 2.1-.6.93L10 10V5Z"/></svg></a>
</div>
</div>
<div class="answercell post-layout--right">
<div class="s-prose js-post-body" itemprop="text">
<pre><code># given random variable X (house price) with population muy = 60, sigma = 40
import scipy as sc
import scipy.stats as sct
sc.version.full_version # 0.15.1
#a. Find P(X&lt;50)
sct.norm.cdf(x=50,loc=60,scale=40) # 0.4012936743170763
#b. Find P(X&gt;=50)
sct.norm.sf(x=50,loc=60,scale=40) # 0.5987063256829237
#c. Find P(60&lt;=X&lt;=80)
sct.norm.cdf(x=80,loc=60,scale=40) - sct.norm.cdf(x=60,loc=60,scale=40)
#d. how much top most 5% expensive house cost at least? or find x where P(X&gt;=x) = 0.05
sct.norm.isf(q=0.05,loc=60,scale=40)
#e. how much top most 5% cheapest house cost at least? or find x where P(X&lt;=x) = 0.05
sct.norm.ppf(q=0.05,loc=60,scale=40)
</code></pre>
</div>
<div class="mt24">
<div class="d-flex fw-wrap ai-start jc-end gs8 gsy">
<time itemprop="dateCreated" datetime="2015-09-26T15:04:38"></time>
<div class="flex--item mr16" style="flex: 1 1 100px;">
<div class="js-post-menu pt2" data-post-id="32798562" data-post-type-id="2">
<div class="d-flex gs8 s-anchors s-anchors__muted fw-wrap">
<div class="flex--item">
<a href="/a/32798562"
rel="nofollow"
itemprop="url"
class="js-share-link js-gps-track"
title="Short permalink to this answer"
data-gps-track="post.click({ item: 2, priv: 0, post_type: 2 })"
data-controller="se-share-sheet"
data-se-share-sheet-title="Share a link to this answer"
data-se-share-sheet-subtitle=""
data-se-share-sheet-post-type="answer"
data-se-share-sheet-social="facebook twitter devto"
data-se-share-sheet-location="2"
data-se-share-sheet-license-url="https%3a%2f%2fcreativecommons.org%2flicenses%2fby-sa%2f3.0%2f"
data-se-share-sheet-license-name="CC BY-SA 3.0"
data-s-popover-placement="bottom-start">Share</a>
</div>
<div class="flex--item">
<button type="button"
id="btnFollowPost-32798562" class="s-btn s-btn__link js-follow-post js-follow-answer js-gps-track"
data-gps-track="post.click({ item: 14, priv: 0, post_type: 2 })"
data-controller="s-tooltip " data-s-tooltip-placement="bottom"
data-s-popover-placement="bottom" aria-controls=""
title="Follow this answer to receive notifications">
Follow
</button>
</div>
</div>
<div class="js-menu-popup-container"></div>
</div>
</div>
<div class="post-signature flex--item fl0">
<div class="user-info ">
<div class="user-action-time">
answered <span title='2015-09-26 15:04:38Z' class='relativetime'>Sep 26, 2015 at 15:04</span>
</div>
<div class="user-gravatar32">
<a href="/users/1125621/o0omycomputero0o"><div class="gravatar-wrapper-32"><img src="https://www.gravatar.com/avatar/c1b3328f06ea4ab079677388658dae10?s=64&amp;d=identicon&amp;r=PG" alt="o0omycomputero0o&#39;s user avatar" width="32" height="32" class="bar-sm"></div></a>
</div>
<div class="user-details" itemprop="author" itemscope itemtype="http://schema.org/Person">
<a href="/users/1125621/o0omycomputero0o">o0omycomputero0o</a><span class="d-none" itemprop="name">o0omycomputero0o</span>
<div class="-flair">
<span class="reputation-score" title="reputation score " dir="ltr">3,376</span><span title="4 gold badges" aria-hidden="true"><span class="badge1"></span><span class="badgecount">4</span></span><span class="v-visible-sr">4 gold badges</span><span title="32 silver badges" aria-hidden="true"><span class="badge2"></span><span class="badgecount">32</span></span><span class="v-visible-sr">32 silver badges</span><span title="45 bronze badges" aria-hidden="true"><span class="badge3"></span><span class="badgecount">45</span></span><span class="v-visible-sr">45 bronze badges</span>
</div>
</div>
</div>
</div>
</div>
</div>
</div>
<span class="d-none" itemprop="commentCount">1</span>
<div class="post-layout--right js-post-comments-component">
<div id="comments-32798562" class="comments js-comments-container bt bc-black-200 mt12 " data-post-id="32798562" data-min-length="15">
<ul class="comments-list js-comments-list"
data-remaining-comments-count="0"
data-canpost="false"
data-cansee="true"
data-comments-unavailable="false"
data-addlink-disabled="true">
<li id="comment-76824582" class="comment js-comment " data-comment-id="76824582" data-comment-owner-id="4815313" data-comment-score="6">
<div class="js-comment-actions comment-actions">
<div class="comment-score js-comment-score js-comment-edit-hide">
<span title="number of &#x27;useful comment&#x27; votes received"
class="warm">6</span>
</div>
</div>
<div class="comment-text js-comment-text-and-form">
<div class="comment-body js-comment-edit-hide">
<span class="comment-copy">PS: You can assume &#39;loc&#39; as &#39;mean&#39; and &#39;scale&#39; as &#39;standard deviation&#39;</span>
<div class="d-inline-flex ai-center">
&ndash;&nbsp;<a href="/users/4815313/suresh2692"
title="3,873 reputation"
class="comment-user">Suresh2692</a>
</div>
<span class="comment-date" dir="ltr"><span title='2017-07-05 11:11:41Z, License: CC BY-SA 3.0' class='relativetime-clean'>Jul 5, 2017 at 11:11</span></span>
</div>
</div>
</li>
</ul>
</div>
<div id="comments-link-32798562" data-rep=50 data-anon=true>
<a class="js-add-link comments-link disabled-link" title="Use comments to ask for more information or suggest improvements. Avoid comments like &#x201C;&#x2B;1&#x201D; or &#x201C;thanks&#x201D;." href="#" role="button">Add a comment</a>
<span class="js-link-separator dno">&nbsp;|&nbsp;</span>
<a class="js-show-link comments-link dno" title="Expand to show all comments on this post" href=# onclick="" role="button"></a>
</div>
</div>
</div>
</div>
<aside class="s-notice s-notice__info post-notice js-post-notice mb16" role="status">
<div class="d-flex fd-column fw-nowrap">
<div class="d-flex fw-nowrap">
<div class="flex--item mr8">
<svg aria-hidden="true" class="svg-icon iconFire" width="18" height="18" viewBox="0 0 18 18"><path fill="#FF6700" d="M13.18 9c-.8.33-1.46.6-1.97 1.3A9.21 9.21 0 0 0 10 13.89a10 10 0 0 0 1.32-.8 2.53 2.53 0 0 1-.63 2.91h.78a3 3 0 0 0 1.66-.5 4.15 4.15 0 0 0 1.26-1.61c.4-.96.47-1.7.55-2.73.05-1.24-.1-2.49-.46-3.68a2 2 0 0 1-.4.91 2.1 2.1 0 0 1-.9.62Z" opacity=".6"/><path fill="#EF2E2E" d="M10.4 12.11a7.1 7.1 0 0 1 .78-1.76c.3-.47.81-.8 1.37-1.08 0 0-.05-3.27-1.55-5.27-1.5-2-3.37-2.75-4.95-2.61 0 0 4.19 2.94 1.18 5.67-2.14 1.92-3.64 3.81-3.1 5.94a4.14 4.14 0 0 0 3.1 3 4.05 4.05 0 0 1 1.08-3.89C9.42 10.92 8 9.79 8 9.79c.67.02 1.3.28 1.81.72a2 2 0 0 1 .58 1.6Z"/></svg>
</div>
<div class="flex--item wmn0 fl1 lh-lg">
<div class="flex--item fl1 lh-lg">
<div>
<b><a href="/help/privileges/protect-questions">Highly active question</a></b>. Earn 10 reputation (not counting the <a href="https://meta.stackexchange.com/questions/141648/what-is-the-association-bonus-and-how-does-it-work">association bonus</a>) in order to answer this question. The reputation requirement helps protect this question from spam and non-answer activity.
</div>
</div>
</div>
</div>
</div>
</aside>
<h2 class="bottom-notice" data-loc="1">
<div>
Not the answer you&#x27;re looking for? Browse other questions tagged <ul class='ml0 list-ls-none js-post-tag-list-wrapper d-inline'><li class='d-inline mr4 js-post-tag-list-item'><a href="/questions/tagged/python" class="post-tag" title="show questions tagged &#39;python&#39;" aria-label="show questions tagged &#39;python&#39;" rel="tag" aria-labelledby="tag-python-tooltip-container">python</a></li><li class='d-inline mr4 js-post-tag-list-item'><a href="/questions/tagged/scipy" class="post-tag" title="show questions tagged &#39;scipy&#39;" aria-label="show questions tagged &#39;scipy&#39;" rel="tag" aria-labelledby="tag-scipy-tooltip-container">scipy</a></li><li class='d-inline mr4 js-post-tag-list-item'><a href="/questions/tagged/normal-distribution" class="post-tag" title="show questions tagged &#39;normal-distribution&#39;" aria-label="show questions tagged &#39;normal-distribution&#39;" rel="tag" aria-labelledby="tag-normal-distribution-tooltip-container">normal-distribution</a></li></ul> or <a href="/questions/ask">ask your own question</a>. </div>
</h2>
</div>
</div>
<div id="sidebar" class="show-votes" role="complementary" aria-label="sidebar">
<div class="s-sidebarwidget s-sidebarwidget__yellow s-anchors s-anchors__grayscale mb16" data-tracker="cb=1">
<ul class="d-block p0 m0">
<li class="s-sidebarwidget--header s-sidebarwidget__small-bold-text d-flex fc-black-500 d:fc-black-600 bb bbw1">
The Overflow Blog
</li>
<li class="s-sidebarwidget--item d-flex px16">
<div class="flex--item1 fl-shrink0">
<svg aria-hidden="true" class="va-text-top svg-icon iconPencilSm" width="14" height="14" viewBox="0 0 14 14"><path fill="#F1B600" d="m2 10.12 6.37-6.43 1.88 1.88L3.88 12H2v-1.88Z"/><path fill="#E87C87" d="m11.1 1.71 1.13 1.12c.2.2.2.51 0 .71L11.1 4.7 9.21 2.86l1.17-1.15c.2-.2.51-.2.71 0Z"/></svg> </div>
<div class="flex--item wmn0 ow-break-word">
<a href="https://stackoverflow.blog/2023/11/29/how-to-scale-a-business-ready-ai-platform-with-watsonx-q-and-a-with-ibm/" class="js-gps-track" data-ga="[&quot;community bulletin board&quot;,&quot;The Overflow Blog&quot;,&quot;https://stackoverflow.blog/2023/11/29/how-to-scale-a-business-ready-ai-platform-with-watsonx-q-and-a-with-ibm/&quot;,null,null]" data-gps-track="communitybulletin.click({ priority: 1, position: 0 })">How to scale a business-ready AI platform with watsonx: Q&amp;A with IBM</a>
<div class="fc-black-400 fs-italic">sponsored post</div>
</div>
</li>
<li class="s-sidebarwidget--item d-flex px16">
<div class="flex--item1 fl-shrink0">
<svg aria-hidden="true" class="va-text-top svg-icon iconPencilSm" width="14" height="14" viewBox="0 0 14 14"><path fill="#F1B600" d="m2 10.12 6.37-6.43 1.88 1.88L3.88 12H2v-1.88Z"/><path fill="#E87C87" d="m11.1 1.71 1.13 1.12c.2.2.2.51 0 .71L11.1 4.7 9.21 2.86l1.17-1.15c.2-.2.51-.2.71 0Z"/></svg> </div>
<div class="flex--item wmn0 ow-break-word">
<a href="https://stackoverflow.blog/2023/12/01/will-developers-return-to-hostile-offices/" class="js-gps-track" data-ga="[&quot;community bulletin board&quot;,&quot;The Overflow Blog&quot;,&quot;https://stackoverflow.blog/2023/12/01/will-developers-return-to-hostile-offices/&quot;,null,null]" data-gps-track="communitybulletin.click({ priority: 1, position: 1 })">Will developers return to hostile offices?</a>
</div>
</li>
<li class="s-sidebarwidget--header s-sidebarwidget__small-bold-text d-flex fc-black-500 d:fc-black-600 bb bbw1">
Featured on Meta
</li>
<li class="s-sidebarwidget--item d-flex px16">
<div class="flex--item1 fl-shrink0">
<div class="favicon favicon-stackexchangemeta" title="Meta Stack Exchange"></div> </div>
<div class="flex--item wmn0 ow-break-word">
<a href="https://meta.stackexchange.com/questions/394860/were-rolling-back-the-changes-to-the-acceptable-use-policy-aup" class="js-gps-track" data-ga="[&quot;community bulletin board&quot;,&quot;Featured on Meta&quot;,&quot;https://meta.stackexchange.com/questions/394860/were-rolling-back-the-changes-to-the-acceptable-use-policy-aup&quot;,null,null]" data-gps-track="communitybulletin.click({ priority: 3, position: 2 })">We&#39;re rolling back the changes to the Acceptable Use Policy (AUP)</a>
</div>
</li>
<li class="s-sidebarwidget--item d-flex px16">
<div class="flex--item1 fl-shrink0">
<div class="favicon favicon-stackexchangemeta" title="Meta Stack Exchange"></div> </div>
<div class="flex--item wmn0 ow-break-word">
<a href="https://meta.stackexchange.com/questions/395062/seeking-feedback-on-tag-colors-update" class="js-gps-track" data-ga="[&quot;community bulletin board&quot;,&quot;Featured on Meta&quot;,&quot;https://meta.stackexchange.com/questions/395062/seeking-feedback-on-tag-colors-update&quot;,null,null]" data-gps-track="communitybulletin.click({ priority: 3, position: 3 })">Seeking feedback on tag colors update</a>
</div>
</li>
<li class="s-sidebarwidget--item d-flex px16">
<div class="flex--item1 fl-shrink0">
<div class="favicon favicon-stackoverflowmeta" title="Meta Stack Overflow"></div> </div>
<div class="flex--item wmn0 ow-break-word">
<a href="https://meta.stackoverflow.com/questions/427199/collectives-updates-new-features-and-ways-to-get-started-with-discussions" class="js-gps-track" data-ga="[&quot;community bulletin board&quot;,&quot;Featured on Meta&quot;,&quot;https://meta.stackoverflow.com/questions/427199/collectives-updates-new-features-and-ways-to-get-started-with-discussions&quot;,null,null]" data-gps-track="communitybulletin.click({ priority: 6, position: 4 })">Collectives updates: new features and ways to get started with Discussions</a>
</div>
</li>
<li class="s-sidebarwidget--item d-flex px16">
<div class="flex--item1 fl-shrink0">
<div class="favicon favicon-stackoverflowmeta" title="Meta Stack Overflow"></div> </div>
<div class="flex--item wmn0 ow-break-word">
<a href="https://meta.stackoverflow.com/questions/427335/overflowai-alpha-invitation-emails-were-distributed-in-error-nov-28th" class="js-gps-track" data-ga="[&quot;community bulletin board&quot;,&quot;Featured on Meta&quot;,&quot;https://meta.stackoverflow.com/questions/427335/overflowai-alpha-invitation-emails-were-distributed-in-error-nov-28th&quot;,null,null]" data-gps-track="communitybulletin.click({ priority: 6, position: 5 })">OverflowAI Alpha invitation emails were distributed in error Nov 28th</a>
</div>
</li>
<li class="s-sidebarwidget--item d-flex px16">
<div class="flex--item1 fl-shrink0">
<div class="favicon favicon-stackoverflowmeta" title="Meta Stack Overflow"></div> </div>
<div class="flex--item wmn0 ow-break-word">
<a href="https://meta.stackoverflow.com/questions/421831/temporary-policy-generative-ai-e-g-chatgpt-is-banned" class="js-gps-track" data-ga="[&quot;community bulletin board&quot;,&quot;Featured on Meta&quot;,&quot;https://meta.stackoverflow.com/questions/421831/temporary-policy-generative-ai-e-g-chatgpt-is-banned&quot;,null,null]" data-gps-track="communitybulletin.click({ priority: 6, position: 6 })">Temporary policy: Generative AI (e.g., ChatGPT) is banned</a>
</div>
</li>
</ul>
</div>
<div class="js-zone-container zone-container-sidebar">
<div id="dfp-tsb" class="everyonelovesstackoverflow everyoneloves__top-sidebar"></div>
<div class="js-report-ad-button-container " style="width: 300px"></div>
</div>
<div class="js-zone-container zone-container-sidebar">
<div id="dfp-msb" class="everyonelovesstackoverflow everyoneloves__mid-sidebar"></div>
<div class="js-report-ad-button-container " style="width: 300px"></div>
</div>
<div id="hireme"></div> <div class="module sidebar-linked">
<h4 id="h-linked">Linked</h4>
<div class="linked" data-tracker="lq=1">
<div class="spacer js-gps-track" data-gps-track="linkedquestion.click({ source_post_id: 20626994, target_question_id: 29369776, position: 0 })">
<a href="/q/29369776" title="Question score (upvotes - downvotes)">
<div class="answer-votes default">9</div>
</a>
<a href="/questions/29369776/what-is-the-sci-numpython-equivalent-to-matlabs-norminv-normal-inverse-cumu?noredirect=1" class="question-hyperlink">What is the [Sci/Num]Python equivalent to Matlabs &quot;norminv&quot; (Normal inverse cumulative distribution function)</a>
</div>
<div class="spacer js-gps-track" data-gps-track="linkedquestion.click({ source_post_id: 20626994, target_question_id: 49115700, position: 1 })">
<a href="/q/49115700" title="Question score (upvotes - downvotes)">
<div class="answer-votes default">0</div>
</a>
<a href="/questions/49115700/python-equivalent-of-matlabs-qfuncinv?noredirect=1" class="question-hyperlink">Python equivalent of MATLAB&#39;s qfuncinv()</a>
</div>
<div class="spacer js-gps-track" data-gps-track="linkedquestion.click({ source_post_id: 20626994, target_question_id: 31648552, position: 2 })">
<a href="/q/31648552" title="Question score (upvotes - downvotes)">
<div class="answer-votes default">0</div>
</a>
<a href="/questions/31648552/how-can-i-get-the-value-in-normal-distribution-in-python?noredirect=1" class="question-hyperlink">How can I get the value in normal distribution in python</a>
</div>
<div class="spacer js-gps-track" data-gps-track="linkedquestion.click({ source_post_id: 20626994, target_question_id: 60699836, position: 3 })">
<a href="/q/60699836" title="Question score (upvotes - downvotes)">
<div class="answer-votes default">24</div>
</a>
<a href="/questions/60699836/how-to-use-norm-ppf?noredirect=1" class="question-hyperlink">How to use norm.ppf()?</a>
</div>
<div class="spacer js-gps-track" data-gps-track="linkedquestion.click({ source_post_id: 20626994, target_question_id: 31910485, position: 4 })">
<a href="/q/31910485" title="Question score (upvotes - downvotes)">
<div class="answer-votes answered-accepted default">9</div>
</a>
<a href="/questions/31910485/quantile-functions-in-python?noredirect=1" class="question-hyperlink">Quantile functions in Python</a>
</div>
<div class="spacer js-gps-track" data-gps-track="linkedquestion.click({ source_post_id: 20626994, target_question_id: 37794648, position: 5 })">
<a href="/q/37794648" title="Question score (upvotes - downvotes)">
<div class="answer-votes default">2</div>
</a>
<a href="/questions/37794648/equivalent-of-matlabs-gaminv-in-python?noredirect=1" class="question-hyperlink">equivalent of Matlab&#39;s gaminv in python</a>
</div>
<div class="spacer js-gps-track" data-gps-track="linkedquestion.click({ source_post_id: 20626994, target_question_id: 75820210, position: 6 })">
<a href="/q/75820210" title="Question score (upvotes - downvotes)">
<div class="answer-votes default">0</div>
</a>
<a href="/questions/75820210/call-scipy-from-matlab-using-python?noredirect=1" class="question-hyperlink">Call SciPy from MATLAB using Python</a>
</div>
<div class="spacer js-gps-track" data-gps-track="linkedquestion.click({ source_post_id: 20626994, target_question_id: 67955360, position: 7 })">
<a href="/q/67955360" title="Question score (upvotes - downvotes)">
<div class="answer-votes default">1</div>
</a>
<a href="/questions/67955360/inverse-of-cumulative-density-function-for-multivariate-normal-distribution?noredirect=1" class="question-hyperlink">Inverse of cumulative density function for Multivariate Normal Distribution</a>
</div>
<div class="spacer js-gps-track" data-gps-track="linkedquestion.click({ source_post_id: 20626994, target_question_id: 49172935, position: 8 })">
<a href="/q/49172935" title="Question score (upvotes - downvotes)">
<div class="answer-votes default">0</div>
</a>
<a href="/questions/49172935/typeerror-ufunc-ndtri-not-supported-for-the-input-types?noredirect=1" class="question-hyperlink">TypeError: ufunc &#39;ndtri&#39; not supported for the input types</a>
</div>
<div class="spacer js-gps-track" data-gps-track="linkedquestion.click({ source_post_id: 20626994, target_question_id: 49045372, position: 9 })">
<a href="/q/49045372" title="Question score (upvotes - downvotes)">
<div class="answer-votes default">0</div>
</a>
<a href="/questions/49045372/how-to-correctly-calculate-the-median-of-a-probability-function?noredirect=1" class="question-hyperlink">How to correctly calculate the MEDIAN of a probability function?</a>
</div>
<div class="spacer more ml32 pl16 pt8">
<a href="/questions/linked/20626994">See more linked questions</a>
</div>
</div>
</div>
<div class="module sidebar-related">
<h4 id="h-related">Related</h4>
<div class="related js-gps-related-questions" data-tracker="rq=3">
<div class="spacer" data-question-id="19589191">
<a href="/q/19589191" title="Question score (upvotes - downvotes)" >
<div class="answer-votes answered-accepted default">16</div>
</a>
<a href="/questions/19589191/the-reverse-inverse-of-the-normal-distribution-function-in-r" class="question-hyperlink">The reverse/inverse of the normal distribution function in R</a>
</div>
<div class="spacer" data-question-id="22264046">
<a href="/q/22264046" title="Question score (upvotes - downvotes)" >
<div class="answer-votes answered-accepted default">5</div>
</a>
<a href="/questions/22264046/inverse-probability-density-function" class="question-hyperlink">Inverse probability density function</a>
</div>
<div class="spacer" data-question-id="37023041">
<a href="/q/37023041" title="Question score (upvotes - downvotes)" >
<div class="answer-votes answered-accepted default">5</div>
</a>
<a href="/questions/37023041/how-to-calculate-the-inverse-of-the-log-normal-cumulative-distribution-function" class="question-hyperlink">How to calculate the inverse of the log normal cumulative distribution function in python?</a>
</div>
<div class="spacer" data-question-id="44983175">
<a href="/q/44983175" title="Question score (upvotes - downvotes)" >
<div class="answer-votes default">2</div>
</a>
<a href="/questions/44983175/calculate-inverse-cdf-from-a-sample-of-data" class="question-hyperlink">Calculate inverse CDF from a sample of data</a>
</div>
<div class="spacer" data-question-id="46199199">
<a href="/q/46199199" title="Question score (upvotes - downvotes)" >
<div class="answer-votes default">0</div>
</a>
<a href="/questions/46199199/how-to-calculate-normal-inverse-cumulative-distribution-function-efficiently-in" class="question-hyperlink">How to calculate normal inverse cumulative distribution function efficiently in Python</a>
</div>
<div class="spacer" data-question-id="47417986">
<a href="/q/47417986" title="Question score (upvotes - downvotes)" >
<div class="answer-votes answered-accepted default">11</div>
</a>
<a href="/questions/47417986/using-scipy-gaussian-kernel-density-estimation-to-calculate-cdf-inverse" class="question-hyperlink">Using scipy gaussian kernel density estimation to calculate CDF inverse</a>
</div>
<div class="spacer" data-question-id="59012763">
<a href="/q/59012763" title="Question score (upvotes - downvotes)" >
<div class="answer-votes default">4</div>
</a>
<a href="/questions/59012763/how-to-calculate-cumulative-normal-distribution-in-python" class="question-hyperlink">How to calculate cumulative normal distribution in python?</a>
</div>
<div class="spacer" data-question-id="60807221">
<a href="/q/60807221" title="Question score (upvotes - downvotes)" >
<div class="answer-votes answered-accepted default">0</div>
</a>
<a href="/questions/60807221/why-not-the-inverse-of-inverse-function-of-a-standard-normal-distribution-calcul" class="question-hyperlink">Why not the inverse of inverse function of a standard normal distribution calculation at scipy.stats python identical?</a>
</div>
<div class="spacer" data-question-id="62899379">
<a href="/q/62899379" title="Question score (upvotes - downvotes)" >
<div class="answer-votes answered-accepted default">0</div>
</a>
<a href="/questions/62899379/inverse-normal-random-number-generation-in-python" class="question-hyperlink">Inverse normal random number generation in python?</a>
</div>
<div class="spacer" data-question-id="70690215">
<a href="/q/70690215" title="Question score (upvotes - downvotes)" >
<div class="answer-votes default">0</div>
</a>
<a href="/questions/70690215/python-reverse-probability-look-up-for-normal-distribution" class="question-hyperlink">python reverse probability look up for normal distribution</a>
</div>
</div>
</div>
<script type="text/javascript">
$(document).ready(function() {
$(".js-gps-related-questions .spacer").click(function () {
fireRelatedEvent($(this).index() + 1, $(this).data('question-id'));
});
function fireRelatedEvent(position, questionId) {
StackExchange.using("gps", function() {
StackExchange.gps.track('related_questions.click',
{
position: position,
originQuestionId: 20626994,
relatedQuestionId: +questionId,
location: 'sidebar',
source: 'Baseline'
});
});
}
});
</script>
<div id="hot-network-questions" class="module tex2jax_ignore">
<h4>
<a href="https://stackexchange.com/questions?tab=hot"
class="js-gps-track s-link s-link__inherit"
data-gps-track="posts_hot_network.click({ item_type:1, location:11 })">
Hot Network Questions
</a>
</h4>
<ul>
<li >
<div class="favicon favicon-unix" title="Unix &amp; Linux Stack Exchange"></div><a href="https://unix.stackexchange.com/questions/762948/how-can-i-reformat-blocks-of-data-until-the-end-of-the-file-is-reached" class="js-gps-track question-hyperlink mb0" data-gps-track="site.switch({ item_type:11, target_site:106 }); posts_hot_network.click({ item_type:2, location:11 })">
How can I reformat blocks of data until the end of the file is reached?
</a>
</li>
<li >
<div class="favicon favicon-politics" title="Politics Stack Exchange"></div><a href="https://politics.stackexchange.com/questions/82982/why-are-there-so-many-pro-palestinian-protestors-in-the-united-states" class="js-gps-track question-hyperlink mb0" data-gps-track="site.switch({ item_type:11, target_site:475 }); posts_hot_network.click({ item_type:2, location:11 })">
Why are there so many pro-Palestinian protestors in the United States?
</a>
</li>
<li >
<div class="favicon favicon-codegolf" title="Code Golf Stack Exchange"></div><a href="https://codegolf.stackexchange.com/questions/267235/ungolf-the-wind" class="js-gps-track question-hyperlink mb0" data-gps-track="site.switch({ item_type:11, target_site:200 }); posts_hot_network.click({ item_type:2, location:11 })">
Ungolf the Wind
</a>
</li>
<li >
<div class="favicon favicon-superuser" title="Super User"></div><a href="https://superuser.com/questions/1819045/high-memory-usage-after-copying-1-million-small-files-win10-x64" class="js-gps-track question-hyperlink mb0" data-gps-track="site.switch({ item_type:11, target_site:3 }); posts_hot_network.click({ item_type:2, location:11 })">
High memory usage after copying 1 million small files (Win10 x64)
</a>
</li>
<li >
<div class="favicon favicon-rpg" title="Role-playing Games Stack Exchange"></div><a href="https://rpg.stackexchange.com/questions/209395/can-you-use-true-polymorph-and-an-intellect-devourer-to-potentially-learn-deep-s" class="js-gps-track question-hyperlink mb0" data-gps-track="site.switch({ item_type:11, target_site:122 }); posts_hot_network.click({ item_type:2, location:11 })">
Can you use True Polymorph and an Intellect Devourer to potentially learn Deep Speech and a secret?
</a>
</li>
<li class="dno js-hidden">
<div class="favicon favicon-codereview" title="Code Review Stack Exchange"></div><a href="https://codereview.stackexchange.com/questions/288197/binary-file-reader-and-writer" class="js-gps-track question-hyperlink mb0" data-gps-track="site.switch({ item_type:11, target_site:196 }); posts_hot_network.click({ item_type:2, location:11 })">
Binary file reader and writer
</a>
</li>
<li class="dno js-hidden">
<div class="favicon favicon-politics" title="Politics Stack Exchange"></div><a href="https://politics.stackexchange.com/questions/82989/are-there-any-wars-other-than-the-2023-israel-hamas-war-that-are-named-after-a-c" class="js-gps-track question-hyperlink mb0" data-gps-track="site.switch({ item_type:11, target_site:475 }); posts_hot_network.click({ item_type:2, location:11 })">
Are there any wars other than the 2023 Israel-Hamas War that are named after a country and a political party?
</a>
</li>
<li class="dno js-hidden">
<div class="favicon favicon-academia" title="Academia Stack Exchange"></div><a href="https://academia.stackexchange.com/questions/204513/supervisor-refuses-to-be-included-as-a-co-author-in-phd-student-papers" class="js-gps-track question-hyperlink mb0" data-gps-track="site.switch({ item_type:11, target_site:415 }); posts_hot_network.click({ item_type:2, location:11 })">
Supervisor refuses to be included as a co-author in PhD student papers?
</a>
</li>
<li class="dno js-hidden">
<div class="favicon favicon-tex" title="TeX - LaTeX Stack Exchange"></div><a href="https://tex.stackexchange.com/questions/702904/how-to-apply-drop-caps-in-verse-environment-in-memoir-class" class="js-gps-track question-hyperlink mb0" data-gps-track="site.switch({ item_type:11, target_site:85 }); posts_hot_network.click({ item_type:2, location:11 })">
How to apply drop caps in verse environment in memoir class?
</a>
</li>
<li class="dno js-hidden">
<div class="favicon favicon-opensource" title="Open Source Stack Exchange"></div><a href="https://opensource.stackexchange.com/questions/14461/distribute-the-code-as-closed-source-and-the-end-users-download-gplv3-dependenci" class="js-gps-track question-hyperlink mb0" data-gps-track="site.switch({ item_type:11, target_site:619 }); posts_hot_network.click({ item_type:2, location:11 })">
Distribute the code as closed source and the end users download GPLv3 dependencies separately
</a>
</li>
<li class="dno js-hidden">
<div class="favicon favicon-scifi" title="Science Fiction &amp; Fantasy Stack Exchange"></div><a href="https://scifi.stackexchange.com/questions/281408/expert-required-to-identify-these-classic-80s-action-movies" class="js-gps-track question-hyperlink mb0" data-gps-track="site.switch({ item_type:11, target_site:186 }); posts_hot_network.click({ item_type:2, location:11 })">
Expert required to identify these classic 80&#x27;s action movies
</a>
</li>
<li class="dno js-hidden">
<div class="favicon favicon-space" title="Space Exploration Stack Exchange"></div><a href="https://space.stackexchange.com/questions/64905/are-there-multiple-types-of-utc-time" class="js-gps-track question-hyperlink mb0" data-gps-track="site.switch({ item_type:11, target_site:508 }); posts_hot_network.click({ item_type:2, location:11 })">
Are there multiple types of UTC time?
</a>
</li>
<li class="dno js-hidden">
<div class="favicon favicon-gaming" title="Arqade"></div><a href="https://gaming.stackexchange.com/questions/405350/who-is-the-small-green-caterpillar-on-my-head" class="js-gps-track question-hyperlink mb0" data-gps-track="site.switch({ item_type:11, target_site:41 }); posts_hot_network.click({ item_type:2, location:11 })">
Who is the small green caterpillar on my head?
</a>
</li>
<li class="dno js-hidden">
<div class="favicon favicon-stackoverflow" title="Stack Overflow"></div><a href="https://stackoverflow.com/questions/77585408/why-do-trigonometric-functions-give-a-seemingly-incorrect-result" class="js-gps-track question-hyperlink mb0" data-gps-track="site.switch({ item_type:11, target_site:1 }); posts_hot_network.click({ item_type:2, location:11 })">
Why do trigonometric functions give a seemingly incorrect result?
</a>
</li>
<li class="dno js-hidden">
<div class="favicon favicon-japanese" title="Japanese Language Stack Exchange"></div><a href="https://japanese.stackexchange.com/questions/101904/does-the-kanji-%e4%b8%a1-have-the-reading-teru-%e3%81%a6%e3%82%8b" class="js-gps-track question-hyperlink mb0" data-gps-track="site.switch({ item_type:11, target_site:257 }); posts_hot_network.click({ item_type:2, location:11 })">
Does the kanji &#x4E21; have the reading teru (&#x3066;&#x308B;)?
</a>
</li>
<li class="dno js-hidden">
<div class="favicon favicon-langdev" title="Programming Language Design and Implementation Stack Exchange"></div><a href="https://langdev.stackexchange.com/questions/3271/why-are-volatile-objects-so-difficult-to-work-with-in-c" class="js-gps-track question-hyperlink mb0" data-gps-track="site.switch({ item_type:11, target_site:716 }); posts_hot_network.click({ item_type:2, location:11 })">
Why are volatile objects so difficult to work with in C&#x2B;&#x2B;?
</a>
</li>
<li class="dno js-hidden">
<div class="favicon favicon-mathoverflow" title="MathOverflow"></div><a href="https://mathoverflow.net/questions/459586/what-are-some-toy-models-for-the-stable-homotopy-groups-of-spheres" class="js-gps-track question-hyperlink mb0" data-gps-track="site.switch({ item_type:11, target_site:504 }); posts_hot_network.click({ item_type:2, location:11 })">
What are some toy models for the stable homotopy groups of spheres?
</a>
</li>
<li class="dno js-hidden">
<div class="favicon favicon-music" title="Music: Practice &amp; Theory Stack Exchange"></div><a href="https://music.stackexchange.com/questions/132863/how-can-i-dampen-the-sound-of-maracas" class="js-gps-track question-hyperlink mb0" data-gps-track="site.switch({ item_type:11, target_site:240 }); posts_hot_network.click({ item_type:2, location:11 })">
How can I dampen the sound of maracas?
</a>
</li>
<li class="dno js-hidden">
<div class="favicon favicon-puzzling" title="Puzzling Stack Exchange"></div><a href="https://puzzling.stackexchange.com/questions/123454/pse-advent-calendar-2023-day-2-wall-i-want-for-christmas" class="js-gps-track question-hyperlink mb0" data-gps-track="site.switch({ item_type:11, target_site:559 }); posts_hot_network.click({ item_type:2, location:11 })">
PSE Advent Calendar 2023 (Day 2): Wall I want for Christmas
</a>
</li>
<li class="dno js-hidden">
<div class="favicon favicon-rpg" title="Role-playing Games Stack Exchange"></div><a href="https://rpg.stackexchange.com/questions/209385/one-of-pcs-backstabbed-a-powerful-ally-how-do-i-punish-them-without-seeming-lik" class="js-gps-track question-hyperlink mb0" data-gps-track="site.switch({ item_type:11, target_site:122 }); posts_hot_network.click({ item_type:2, location:11 })">
One of PCs backstabbed a powerful ally. How do I punish them without seeming like singling them out?
</a>
</li>
<li class="dno js-hidden">
<div class="favicon favicon-workplace" title="The Workplace Stack Exchange"></div><a href="https://workplace.stackexchange.com/questions/194505/how-does-one-get-past-unlucky-work-history-streak" class="js-gps-track question-hyperlink mb0" data-gps-track="site.switch({ item_type:11, target_site:423 }); posts_hot_network.click({ item_type:2, location:11 })">
How does one get past unlucky work history streak?
</a>
</li>
<li class="dno js-hidden">
<div class="favicon favicon-tex" title="TeX - LaTeX Stack Exchange"></div><a href="https://tex.stackexchange.com/questions/702915/how-to-handle-urls-with-tex-special-charactors-in-a-footnote" class="js-gps-track question-hyperlink mb0" data-gps-track="site.switch({ item_type:11, target_site:85 }); posts_hot_network.click({ item_type:2, location:11 })">
How to handle URLs with TeX-special charactors in a footnote
</a>
</li>
<li class="dno js-hidden">
<div class="favicon favicon-crafts" title="Arts &amp; Crafts Stack Exchange"></div><a href="https://crafts.stackexchange.com/questions/11981/is-there-an-easily-accessible-way-to-make-a-surface-very-bright-white" class="js-gps-track question-hyperlink mb0" data-gps-track="site.switch({ item_type:11, target_site:650 }); posts_hot_network.click({ item_type:2, location:11 })">
Is there an easily accessible way to make a surface very bright white?
</a>
</li>
<li class="dno js-hidden">
<div class="favicon favicon-academia" title="Academia Stack Exchange"></div><a href="https://academia.stackexchange.com/questions/204530/advisor-student-collaboration-or-lack-thereof-and-paper-authorship-in-mathemat" class="js-gps-track question-hyperlink mb0" data-gps-track="site.switch({ item_type:11, target_site:415 }); posts_hot_network.click({ item_type:2, location:11 })">
Advisor-student collaboration (or lack thereof) and paper authorship in mathematics: how does it work?
</a>
</li>
</ul>
<a href="#"
class="show-more js-show-more js-gps-track"
data-gps-track="posts_hot_network.click({ item_type:3, location:11 })">
more hot questions
</a>
</div>
<div id="feed-link" class="js-feed-link">
<a href="/feeds/question/20626994" title="Feed of this question and its answers">
<svg aria-hidden="true" class="fc-orange-400 svg-icon iconRss" width="18" height="18" viewBox="0 0 18 18"><path d="M3 1a2 2 0 0 0-2 2v12c0 1.1.9 2 2 2h12a2 2 0 0 0 2-2V3a2 2 0 0 0-2-2H3Zm0 1.5c6.9 0 12.5 5.6 12.5 12.5H13C13 9.55 8.45 5 3 5V2.5Zm0 5c4.08 0 7.5 3.41 7.5 7.5H8c0-2.72-2.28-5-5-5V7.5Zm0 5c1.36 0 2.5 1.14 2.5 2.5H3v-2.5Z"/></svg>
Question feed
</a>
</div>
<aside class="s-modal js-feed-link-modal" tabindex="-1" role="dialog" aria-labelledby="feed-modal-title" aria-describedby="feed-modal-description" aria-hidden="true">
<div class="s-modal--dialog js-modal-dialog wmx4" role="document" data-controller="se-draggable">
<h1 class="s-modal--header fw-bold js-first-tabbable" id="feed-modal-title" data-se-draggable-target="handle" tabindex="0">
Subscribe to RSS
</h1>
<div class="d-flex gs4 gsy fd-column">
<div class="flex--item">
<label class="d-block s-label c-default" for="feed-url">
Question feed
<p class="s-description mt2" id="feed-modal-description">To subscribe to this RSS feed, copy and paste this URL into your RSS reader.</p>
</label>
</div>
<div class="d-flex ps-relative">
<input class="s-input" type="text" name="feed-url" id="feed-url" readonly="readonly" value="https://stackoverflow.com/feeds/question/20626994" />
<svg aria-hidden="true" class="s-input-icon fc-orange-400 svg-icon iconRss" width="18" height="18" viewBox="0 0 18 18"><path d="M3 1a2 2 0 0 0-2 2v12c0 1.1.9 2 2 2h12a2 2 0 0 0 2-2V3a2 2 0 0 0-2-2H3Zm0 1.5c6.9 0 12.5 5.6 12.5 12.5H13C13 9.55 8.45 5 3 5V2.5Zm0 5c4.08 0 7.5 3.41 7.5 7.5H8c0-2.72-2.28-5-5-5V7.5Zm0 5c1.36 0 2.5 1.14 2.5 2.5H3v-2.5Z"/></svg>
</div>
</div>
<a class="s-modal--close s-btn s-btn__muted js-modal-close js-last-tabbable" href="#" aria-label="Close">
<svg aria-hidden="true" class="svg-icon iconClearSm" width="14" height="14" viewBox="0 0 14 14"><path d="M12 3.41 10.59 2 7 5.59 3.41 2 2 3.41 5.59 7 2 10.59 3.41 12 7 8.41 10.59 12 12 10.59 8.41 7 12 3.41Z"/></svg>
</a>
</div>
</aside>
</div>
</div>
<script>StackExchange.ready(function(){$.get('/posts/20626994/ivc/44b3?prg=70cd82cb-cc0a-4f50-a471-7090a00d40da');});</script>
<noscript><div><img src="/posts/20626994/ivc/44b3?prg=70cd82cb-cc0a-4f50-a471-7090a00d40da" class="dno" alt="" width="0" height="0"></div></noscript><div style="display:none" id="js-codeblock-lang">lang-py</div></div>
</div>
</div>
<script type="text/javascript">
var cam = cam || { opt: {} };
var clcGamLoaderOptions = cam || { opt: {} };
var opt = clcGamLoaderOptions.opt;
opt.omni = 'BwoLCMyGlY6Jjro8EAUYsvzqCSACKAI6InxweXRob258c2NpcHl8bm9ybWFsLWRpc3RyaWJ1dGlvbnxIAPIF59cxX0wgerk';
opt.refresh = !1;
opt.refreshInterval = 90;
opt.sf = !0;
opt.hb = !1;
opt.ll = !0;
opt.tlb_position = 0;
opt.personalization_consent = !1;
opt.targeting_consent = !1;
opt.performance_consent = !1;
opt.targeting = {Registered:['false'],'so-tag':['python','scipy','normal-distribution'],'tag-reportable':['python','scipy','normal-distribution'],NumberOfAnswers:['3']};
opt.adReportEnabled = !0;
opt.adReportUrl = '/ads/report-ad';
opt.adReportText = 'Report this ad';
opt.adReportFileTypeErrorMessage = 'Please select a PNG or JPG file.';
opt.adReportFileSizeErrorMessage = 'The file must be under 2 MiB.';
opt.adReportErrorText = 'Error uploading ad report.';
opt.adReportThanksText = 'Thanks for your feedback. Well review this against our code of conduct and take action if necessary.';
opt.adReportLoginExpiredMessage = 'Your login session has expired, please login and try again.';
opt.adReportLoginErrorMessage = 'An error occurred when loading the report form - please try again';
opt.adReportModalClass = 'js-ad-report';
opt.perRequestGuid = '70cd82cb-cc0a-4f50-a471-7090a00d40da';
opt.responseHash = 'wFWdf7nbguQxGgmc6Xi6549SI7TAcptfAqv01Ih/NvA=';
opt.targeting.TargetingConsent = ['False_Passive'];
const urlParams = new URLSearchParams(window.location.search);
if (urlParams.has('dfptestads')) {
const dfptestads = urlParams.get('dfptestads');
opt.targeting.DfpTestAds = dfptestads;
}
</script>
<script>;(()=>{"use strict";var __webpack_modules__={23:(e,t,s)=>{s.d(t,{Z7:()=>d,eq:()=>a,kG:()=>r});const n=/^\/tags\//.test(location.pathname)||/^\/questions\/tagged\//.test(location.pathname)?"tag-pages":/^\/$/.test(location.pathname)||/^\/home/.test(location.pathname)?"home-page":"question-pages";let o=location.hostname;const i={slots:{lb:[[728,90]],mlb:[[728,90]],smlb:[[728,90]],bmlb:[[728,90]],sb:e=>"dfp-tsb"===e?[[300,250],[300,600]]:[[300,250]],"tag-sponsorship":[[730,135]],"mobile-below-question":[[320,50],[300,250]],msb:[[300,250],[300,600]],"talent-conversion-tracking":[[1,1]],"site-sponsorship":[[230,60]]},ids:{"dfp-tlb":"lb","dfp-mlb":"mlb","dfp-smlb":"smlb","dfp-bmlb":"bmlb","dfp-tsb":"sb","dfp-isb":"sb","dfp-tag":"tag-sponsorship","dfp-msb":"msb","dfp-sspon":"site-sponsorship","dfp-m-aq":"mobile-below-question"},idsToExcludeFromAdReports:["dfp-sspon"]};function r(){return Object.keys(i.ids)}function a(e){return i.idsToExcludeFromAdReports.indexOf(e)<0}function d(e){var t=e.split("_")[0];const s=i.ids[t];let r=i.slots[s];return"function"==typeof r&&(r=r(t)),{path:`/248424177/${o}/${s}/${n}`,sizes:r,zone:s}}},865:(e,t,s)=>{function n(e){return"string"==typeof e?document.getElementById(e):e}function o(e){return!!(e=n(e))&&"none"===getComputedStyle(e).display}function i(e){return!o(e)}function r(e){return!!e}function a(e){return/^\s*$/.test(n(e).innerHTML)}function d(e){const{style:t}=e;t.height=t.maxHeight=t.minHeight="auto",t.display="none"}function l(e){const{style:t}=e;t.height=t.maxHeight=t.minHeight="auto",t.display="none",[].forEach.call(e.children,l)}function c(e){const{style:t}=e;t.height=t.maxHeight=t.minHeight="auto",t.removeProperty("display")}function g(e){const t=document.createElement("script");t.src=e,document.body.appendChild(t)}function p(e){return s=e,(t=[]).push=function(e){return s(),delete this.push,this.push(e)},t;var t,s}function h(e){let t="function"==typeof HTMLTemplateElement;var s=document.createElement(t?"template":"div");return e=e.trim(),s.innerHTML=e,t?s.content.firstChild:s.firstChild}s.d(t,{$Z:()=>c,Bv:()=>h,Gx:()=>g,Nj:()=>n,QZ:()=>p,cf:()=>d,pn:()=>i,wo:()=>l,xb:()=>a,xj:()=>o,yb:()=>r})},763:(__unused_webpack_module,__webpack_exports__,__webpack_require__)=>{__webpack_require__.d(__webpack_exports__,{t:()=>AdReports});var _common_helper__WEBPACK_IMPORTED_MODULE_2__=__webpack_require__(865),_console__WEBPACK_IMPORTED_MODULE_1__=__webpack_require__(276),_ad_units__WEBPACK_IMPORTED_MODULE_0__=__webpack_require__(23);class AdReports{constructor(e,t){if(this.googletag=e,this.cam=t,this.allowedFileTypes=["image/png","image/jpg","image/jpeg"],this.ignoreValidation=!1,_console__WEBPACK_IMPORTED_MODULE_1__.cM("Ad reporting init"),this.cam=t,this.callOnButtonClick=e=>this.onButtonClick(e),this.googletag.pubads().addEventListener("slotRenderEnded",e=>this.handleSlotRendered(e)),Array.isArray(t.slotsRenderedEvents)){_console__WEBPACK_IMPORTED_MODULE_1__.cM("Adding report button to "+t.slotsRenderedEvents.length+" events that have transpired");for(var s=0;s<t.slotsRenderedEvents.length;s++)this.handleSlotRendered(t.slotsRenderedEvents[s])}}handleSlotRendered(e){if(e&&e.slot&&!e.isEmpty&&(e.creativeId||e.lineItemId||!e.isEmpty)){var t=e.slot.getSlotElementId();if(t){var s=document.getElementById(t);if(s)if((0,_ad_units__WEBPACK_IMPORTED_MODULE_0__.eq)(t)){var n=s?.closest(".js-zone-container")?.querySelector(".js-report-ad-button-container");n.innerHTML="",n.append(this.createButton(e)),n.style.height="24px",_console__WEBPACK_IMPORTED_MODULE_1__.cM("Added report button to the bottom of "+t)}else _console__WEBPACK_IMPORTED_MODULE_1__.cM("Not adding report button to the bottom of "+t+": shouldHaveReportButton = false");else _console__WEBPACK_IMPORTED_MODULE_1__.cM("Not adding report button to the bottom of "+t+": resolved invalid adUnit element")}else _console__WEBPACK_IMPORTED_MODULE_1__.cM("Not adding report button to the bottom of element: invalid adUnitElementId")}else _console__WEBPACK_IMPORTED_MODULE_1__.cM("Not adding report button to the bottom of element: invalid SlotRenderEndedEvent")}async onButtonClick(e){e.preventDefault();let t=e.target;const s=t.dataset.modalUrl,n=t.dataset.googleEventData;return await this.loadModal(s,t,n),!1}createButton(e){let t=document.createElement("button");var s=JSON.stringify(e);return t.dataset.googleEventData=s,t.dataset.modalUrl=this.cam.opt.adReportUrl,t.dataset.adUnit=e.slot.getSlotElementId(),t.classList.add("js-report-ad","s-btn","s-btn__link","fs-fine","mt2","float-right"),t.append(document.createTextNode(this.cam.opt.adReportText)),t.removeEventListener("click",this.callOnButtonClick),t.addEventListener("click",this.callOnButtonClick),t}async loadModal(url,$link,googleEventData){try{await window.StackExchange.helpers.loadModal(url,{returnElements:window.$($link)}),this.initForm(googleEventData)}catch(e){var message="",response=e.responseText?eval(`(${e.responseText})`):null;message=response&&response.isLoggedOut?this.cam.opt.adReportLoginExpiredMessage:this.cam.opt.adReportLoginErrorMessage,window.StackExchange.helpers.showToast(message,{type:"danger"})}}removeModal(){window.StackExchange.helpers.closePopups(document.querySelectorAll("."+this.cam.opt.adReportModalClass),"dismiss")}initForm(e,t=!1){this.ignoreValidation=t,this.$form=document.querySelector(".js-ad-report-form"),this.$googleEventData=this.$form.querySelector(".js-json-data"),this.$adReportReasons=this.$form.querySelectorAll(".js-ad-report-reason"),this.$adReportReasonOther=this.$form.querySelector(".js-ad-report-reason-other"),this.$fileUploaderInput=this.$form.querySelector(".js-file-uploader-input"),this.$imageUploader=this.$form.querySelector(".js-image-uploader"),this.$clearImageUpload=this.$form.querySelector(".js-clear-image-upload"),this.$imageUploaderText=this.$form.querySelector(".js-image-uploader-text"),this.$imageUploaderPreview=this.$form.querySelector(".js-image-uploader-preview"),this.$fileErrorMessage=this.$form.querySelector(".js-file-error");const s=this.$form.querySelector(".js-drag-drop-enabled"),n=this.$form.querySelector(".js-drag-drop-disabled");this.$googleEventData.value=e,this.$adReportReasons.forEach((e,t)=>e.addEventListener("change",e=>{this.$adReportReasonOther.classList.toggle("d-none","3"!==e.target.value)})),this.$fileUploaderInput.addEventListener("change",()=>{this.validateFileInput()&&this.updateImagePreview(this.$fileUploaderInput.files)}),this.$clearImageUpload.addEventListener("click",e=>{e.preventDefault(),this.clearImageUpload()});try{this.$fileUploaderInput[0].value="",this.$imageUploader.addEventListener("dragenter dragover dragleave drop",this.preventDefaults),this.$imageUploader.addEventListener("dragenter dragover",this.handleDragStart),this.$imageUploader.addEventListener("dragleave drop",this.handleDragEnd),this.$imageUploader.addEventListener("drop",this.handleDrop)}catch(e){s.classList.add("d-none"),n.classList.remove("d-none")}this.$form.removeEventListener("",this.handleDragEnd),this.$form.addEventListener("submit",async e=>(e.preventDefault(),this.submitForm(),!1))}clearImageUpload(){this.$fileUploaderInput.value="",this.$imageUploaderPreview.setAttribute("src",""),this.$imageUploaderPreview.classList.add("d-none"),this.$clearImageUpload.classList.add("d-none"),this.$imageUploaderText.classList.remove("d-none"),this.$imageUploader.classList.add("p16","ba","bas-dashed","bc-black-100")}preventDefaults(e){e.preventDefault(),e.stopPropagation()}handleDragStart(e){this.$imageUploader.classList.remove("bas-dashed"),this.$imageUploader.classList.add("bas-solid","bc-black-100")}handleDragEnd(e){this.$imageUploader.classList.remove("bas-solid","bc-black-100"),this.$imageUploader.classList.add("bas-dashed")}handleDrop(e){var t=e.originalEvent.dataTransfer.files;FileReader&&t&&1===t.length&&(this.$fileUploaderInput.files=t,this.validateFileInput()&&this.updateImagePreview(t))}setError(e){this.$fileErrorMessage.parentElement.classList.toggle("has-error",e)}updateImagePreview(e){this.$imageUploader.classList.remove("p16","ba","bas-dashed","bc-black-100"),this.$clearImageUpload.classList.remove("d-none"),this.$imageUploaderText.classList.add("d-none");var t=new FileReader;t.onload=e=>{null!=e.target&&(this.$imageUploaderPreview.setAttribute("src",e.target.result),this.$imageUploaderPreview.classList.remove("d-none"))},t.readAsDataURL(e[0])}validateFileInput(){if(this.ignoreValidation)return!0;const e=this.cam.opt.adReportFileTypeErrorMessage,t=this.cam.opt.adReportFileSizeErrorMessage;if(null==this.$fileUploaderInput.files)return!1;var s=this.$fileUploaderInput.files[0];return null==s?(this.setError(!0),!1):this.allowedFileTypes.indexOf(s.type)<0?(this.$fileErrorMessage.textContent=e,this.$fileErrorMessage.classList.remove("d-none"),this.setError(!0),!1):s.size>2097152?(this.$fileErrorMessage.textContent=t,this.$fileErrorMessage.classList.remove("d-none"),this.setError(!0),!1):(this.$fileErrorMessage.classList.add("d-none"),this.setError(!1),!0)}async gatherDiagnosticInfo(){return{BrowserVersion:await this.getBrowserVersion()}}getElementSource(e){return e.outerHTML}getNestedIFrameElement(e){var t=e.querySelector("iframe");return t.contentDocument?t.contentDocument.documentElement:t.contentWindow.document.documentElement}async getBrowserVersion(){return await navigator.userAgentData.getHighEntropyValues(["fullVersionList"]).then(e=>JSON.stringify(e.fullVersionList))}async submitForm(){if(!this.validateFileInput())return!1;this.$form.querySelector("[type=submit]").setAttribute("disabled","true");var e=JSON.parse(this.$googleEventData.value||"{}");e.Reason=parseInt(this.$form.querySelector(".js-ad-report-reason:checked").value,10),e.Description=this.$adReportReasonOther.value,this.$googleEventData.value=JSON.stringify(e);var t=new FormData(this.$form);if("1"===t.get("shareDiagnosticInfo")){var s=await this.gatherDiagnosticInfo();Object.keys(s).forEach(e=>t.append(e,s[e]))}try{const e=await window.fetch(this.$form.getAttribute("action"),{method:this.$form.getAttribute("method"),body:t,cache:"no-cache"}),s=e.headers.get("content-type")||"",o=await e.text();if(!e.ok)throw new Error("response not valid");if(0===s.indexOf("text/html")){var n=(0,_common_helper__WEBPACK_IMPORTED_MODULE_2__.Bv)(o);const e=n?n.querySelector(".js-modal-content"):null;if(_console__WEBPACK_IMPORTED_MODULE_1__.cM("$popupContent"),_console__WEBPACK_IMPORTED_MODULE_1__.cM(e),!e)throw new Error(`Could not find .js-modal-content in response from ${this.$form.getAttribute("action")}`);document.querySelector(".js-modal-content").replaceWith(e)}else window.StackExchange.helpers.showToast(this.cam.opt.adReportThanksText,{type:"success"}),this.removeModal()}catch(e){window.StackExchange.helpers.showToast(this.cam.opt.adReportErrorText,{type:"danger"})}finally{let e=this.$form.querySelector("[type=submit]");e&&e.removeAttribute("disabled")}}}},276:(e,t,s)=>{function n(...e){}function o(...e){}s.d(t,{cM:()=>n,vU:()=>o})}},__webpack_module_cache__={};function __webpack_require__(e){var t=__webpack_module_cache__[e];if(void 0!==t)return t.exports;var s=__webpack_module_cache__[e]={exports:{}};return __webpack_modules__[e](s,s.exports,__webpack_require__),s.exports}__webpack_require__.d=(e,t)=>{for(var s in t)__webpack_require__.o(t,s)&&!__webpack_require__.o(e,s)&&Object.defineProperty(e,s,{enumerable:!0,get:t[s]})},__webpack_require__.o=(e,t)=>Object.prototype.hasOwnProperty.call(e,t);var __webpack_exports__={};(()=>{var e=__webpack_require__(276),t=(e=>(e[e.Above=0]="Above",e[e.Below=1]="Below",e))(t||{});const s=Object.assign({},{"lib":"https://cdn.sstatic.net/clc/js/bundles/gam_loader_script/gam_loader_script.bundle.741.844f6c3cc28ce2d65ae9.min.js","style":null,"u":null,"wa":true,"kt":2000,"tto":true,"h":"clc.stackoverflow.com","allowed":"^(((talent\\.)?stackoverflow)|(blog\\.codinghorror)|(.*\\.googlesyndication)|(serverfault|askubuntu)|([^\\.]+\\.stackexchange))\\.com$","wv":true,"al":false,"abd":true,"cpa_liid":[5882654614],"cpa_cid":[138377597667],"dp":false,"tgt_to":1000,"tgt_u":"https://clc.stackoverflow.com/get-user-acct-tgt","tgt_e":false,"tgt_p":0});var n=__webpack_require__(23),o=__webpack_require__(865),i=__webpack_require__(763);class r{constructor(t,s){this.googletag=t,this.interval=s,e.cM("Ad refresh init. interval: "+s),this.googletag.pubads().addEventListener("impressionViewable",e=>this.onImpressionViewable(e)),e.cM("done enabling ad refresh")}onImpressionViewable(t){var s=t.slot;e.cM("ad refresh - slot "+s.getSlotElementId()+" is viewable, initializing refresh"),this.scheduleRefresh(s)}scheduleRefresh(e){setTimeout(()=>this.refreshAdSlot(e),1e3*this.interval)}static refreshMyAd(t,s){let n=t.pubads().getSlots().find(e=>e.getSlotElementId()===s);n&&(e.cM("refreshMyAd - refreshing ad slot "+s),t.pubads().refresh([n]))}refreshAdSlot(t){var s=t.getSlotElementId();this.isElementVisibleInBrowser(s)?(e.cM("refreshing ad slot "+s),googletag.pubads().refresh([t])):(e.cM("refresh skipped this time; ad slot not viewable:"+s),this.scheduleRefresh(t))}isElementVisibleInBrowser(e){var t=document.getElementById(e);if(null!==t){var s=t.getBoundingClientRect();if(s.top>=0&&s.left>=0&&s.bottom<=(window.innerHeight||document.documentElement.clientHeight)&&s.right<=(window.innerWidth||document.documentElement.clientWidth))return!0}return!1}}var a=(e=>(e.Off="Off",e.PreSurvey="PreSurvey",e.Collect="Collect",e.PostSurvey="PostSurvey",e))(a||{});class d{constructor(e,t){this.lineItemImpressions=[],this.surveysIdsCompleted=[],this.lineItemImpressions=e,this.surveysIdsCompleted=t}addImpression(e,t){let s={brandId:e,lineItemId:t,timestamp:new Date};this.lineItemImpressions.push(s)}addBrandSurveyCompleted(e){-1===this.surveysIdsCompleted.indexOf(e)&&this.surveysIdsCompleted.push(e)}getTotalBrandImpressions(){let e=new Map;for(let t of this.lineItemImpressions)if(e.has(t.brandId)){let s=e.get(t.brandId);e.set(t.brandId,s+1)}else e.set(t.brandId,1);return e}getBrandLineItemImpressions(e){let t={};for(let s of this.lineItemImpressions)if(s.brandId==e)if(void 0!==t[s.lineItemId]){let e=t[s.lineItemId];t[s.lineItemId]=e+1}else t[s.lineItemId]=1;return t}}class l{constructor(){this.surveyEngagementLocalStorageKey="clc-survey-engagement"}getBrandSurveyEngagement(){let e=localStorage.getItem(this.surveyEngagementLocalStorageKey);if(null===e)return new d([],[]);let t=JSON.parse(e);return new d(t.lineItemImpressions,t.surveysIdsCompleted)}saveBrandSurveyEngagement(e){let t=JSON.stringify(e);localStorage.setItem(this.surveyEngagementLocalStorageKey,t)}}class c{constructor(){this.surveyRepository=new l}getBrandSurveyEngagement(){return this.surveyRepository.getBrandSurveyEngagement()}recordImpression(e,t){let s=this.getBrandSurveyEngagement();s.addImpression(e,t),this.surveyRepository.saveBrandSurveyEngagement(s)}recordBrandSurveyCompleted(e){let t=this.getBrandSurveyEngagement();t.addBrandSurveyCompleted(e),this.surveyRepository.saveBrandSurveyEngagement(t)}}class g{constructor(t,s){this.googletag=t,this.brandSettings=s,this.brandSlotMap=new Map,this.brandSurveyEngagementService=new c,e.cM("Brand Survey init: "+JSON.stringify(s)),void 0!==s?(this.googletag.pubads().addEventListener("slotRenderEnded",e=>this.handleSlotRendered(e)),this.googletag.pubads().addEventListener("impressionViewable",e=>this.onImpressionViewable(e)),e.cM("done enabling Brand Survey")):e.cM("Brand Survey init: brandSettings is undefined, not initializing")}handleSlotRendered(t){e.cM("Brand Survey - slot rendered - slot:"+JSON.stringify(t.slot.getSlotElementId())+" lineItem: "+t.lineItemId);let s=this.findItemWithId(t.lineItemId);if(null===s||s.mode!==a.Collect)this.brandSlotMap.delete(t.slot.getSlotElementId());else{let e={brandId:s.brandId,lineItemId:t.lineItemId};this.brandSlotMap.set(t.slot.getSlotElementId(),e)}}onImpressionViewable(t){let s=t.slot;if(e.cM("ad - Brand Survey - impression viewable. Details: "+JSON.stringify(s.getSlotElementId())),e.cM("ad - Brand Survey - slot "+s.getSlotElementId()+" is viewable"),this.brandSlotMap.has(s.getSlotElementId())){let t=this.brandSlotMap.get(s.getSlotElementId());e.cM("Brand Survey - brand "+t.brandId+" is viewable"),this.recordImpression(this.brandSlotMap.get(s.getSlotElementId()))}}recordImpression(t){e.cM("ad - Brand Survey - recording impression for brand "+t.brandId),this.brandSurveyEngagementService.recordImpression(t.brandId,t.lineItemId)}findItemWithId(t){return e.cM("brand settings: "+JSON.stringify(this.brandSettings)),this.brandSettings.find(e=>e.lineItemIds.includes(t))||null}}const p="response-brand-survey-submit|",h="request-brand-survey-metadata|",m="record-metric-on-server|",u="request-dsp-tags",f="response-dsp-tags|";class v{static refreshAdIfBrandSurveyIsDuplicated(e,t,s){this.alreadyCompletedThisBrandSurvey(t)&&r.refreshMyAd(e,s)}static alreadyCompletedThisBrandSurvey(e){return(new c).getBrandSurveyEngagement().surveysIdsCompleted.includes(e)}}window.cam=new class{constructor(t=null){if(this.gptImported=!1,this.slotsRenderedEvents=[],this.collapsed={},e.cM("constructor"),this.clc_options=s,window.clcGamLoaderOptions)Object.assign(this,window.clcGamLoaderOptions);else if(void 0===this.opt){let e=window.opt;e&&(this.opt=e)}}init(){if(e.cM("init"),void 0===this.opt)throw new Error("opt not set, required by GAM Loader");e.cM("init brand survey service"),this.getUserAccountTargetingPromise=this.getUserAccountTargeting(),e.cM("setup message handler"),window.addEventListener("message",e=>{this.onmessage(e)})}handleSlotRenderedNoAdReport(){if(googletag.pubads().addEventListener("slotRenderEnded",e=>this.applyExtraMarginBottom(e)),Array.isArray(this.slotsRenderedEvents))for(var e=0;e<this.slotsRenderedEvents.length;e++)this.applyExtraMarginBottom(this.slotsRenderedEvents[e])}onmessage(t){let s="omni";if(t.data&&("string"==typeof t.data||t.data instanceof String))if(0===t.data.indexOf("get-omni-")){e.cM("Recevied get-omni message, sending back omni");var n=t.source,i=this.opt.omni,r="string"==typeof i?i:"";n.postMessage([s,r,this.opt.perRequestGuid].join("|"),"*")}else if(0===t.data.indexOf("collapse-")){e.cM("Recevied collapse message, collapse ad iframe"),e.cM(t);for(var a=t.source.window,d=document.getElementsByTagName("IFRAME"),l=0;l<d.length;l++){var g=d[l];if(g.contentWindow==a)return void(0,o.wo)(g.parentElement.parentElement.parentElement)}}else if(0===t.data.indexOf("resize|")){e.cM("Recevied resize message, resize ad iframe"),e.cM(t);let s=this._getFrameByEvent(t),n=t.data.indexOf("|")+1,o=t.data.slice(n),i=parseFloat(o)+.5;e.cM("New iframe height "+i),s.height=i.toString(),s.parentElement.style.height=i.toString()+"px"}else if(0===t.data.indexOf("getmarkup|")){let s=t.data.indexOf("|")+1,n=t.data.slice(s);e.cM("Recevied get markup message: "+n);let o=this._getFrameByEvent(t).closest(".everyonelovesstackoverflow");const i=document.createElement("script");i.dataset.adZoneId=o.id,i.src=n,document.body.appendChild(i)}else if(0===t.data.indexOf("window-location|")){let s=t.data.indexOf("|")+1,n=t.data.slice(s);e.cM("Recevied window location message: "+n),n.startsWith("/")||(n="/"+n),window.open(window.location.protocol+"//"+window.location.host+n,"_blank")}else if(0===t.data.indexOf("request-brand-survey-submit|")){let s=t.data.split("|"),n=s[1],o=s[2],i=s[3],r=JSON.parse(i);e.cM(n),e.cM(o),e.cM(i),e.cM("Received brand survey "+n+" response message: "+o);var b=new FormData;for(var _ in r)b.append(_,r[_]);let a=this._getFrameByEvent(t);if(v.alreadyCompletedThisBrandSurvey(+n))return e.cM("Already completed this brand survey. Not submitting duplicate to server."),void a.contentWindow.postMessage("response-brand-survey-submit-duplicate|","*");e.cM("Send the brand survey to the server"),fetch(o,{method:"POST",body:b}).then(e=>e.json()).then(e=>a.contentWindow.postMessage(p,"*")).catch(e=>a.contentWindow.postMessage(p,"*"))}else if(0===t.data.indexOf("brand-survey-completed-store|")){let s=t.data.split("|"),n=(s[1],s[2]);if(e.cM("Received brand survey completed store message for survey ID "+n),v.alreadyCompletedThisBrandSurvey(+n))return void e.cM("Already completed this brand survey. Not recording duplicate locally.");e.cM("Record brand survey completion locally"),(new c).recordBrandSurveyCompleted(+n)}else if(0===t.data.indexOf(h)){let s=t.data.split("|"),n=s[1],o=s[2];e.cM("Received message: request-brand-survey-metadata| with Brand Survey ID "+o);let i=(new c).getBrandSurveyEngagement().getBrandLineItemImpressions(+n),r=JSON.stringify(i),a=this._getFrameByEvent(t);e.cM("sending impression data: "+r),a.contentWindow.postMessage("response-brand-survey-metadata|"+this.opt.responseHash+"|"+this.opt.perRequestGuid+"|"+r,"*")}else if(0===t.data.indexOf("refresh-if-duplicate-brand-survey|")){let e=t.data.split("|")[1],s=this.getSlotElementIdByEvent(t);v.refreshAdIfBrandSurveyIsDuplicated(googletag,+e,s)}else if(0===t.data.indexOf(m)){e.cM("Received message: record-metric-on-server| with args: "+t.data);let s=t.data.split("|"),n=s[1],o=s[2],i=s[3],r=s[4],a=new FormData;a.append("brandSurveyId",i.toString()),a.append("responseHash",this.opt.responseHash),a.append("perRequestGuid",this.opt.perRequestGuid),a.append("questionNumber",n.toString()),a.append("metricType",r.toString()),fetch(o,{method:"POST",body:a}).then(e=>e.ok).catch(t=>{e.cM("SendMetricToServer: Error sending metric to server: "+t)})}else if(0===t.data.indexOf(u)){e.cM("Received message: request-dsp-tags with args: "+t.data);let s=this._getFrameByEvent(t);if(!this.opt.targeting["so-tag"])return void s.contentWindow.postMessage(f,"*");const n=this.opt.targeting["so-tag"].join(",");e.cM("sending targeting tags: "+n),s.contentWindow.postMessage(f+n,"*")}else e.cM("Received unhandled message")}getSlotElementIdByEvent(e){let t=this._getFrameByEvent(e).parentElement?.parentElement?.id;return t||""}_getFrameByEvent(e){return Array.from(document.getElementsByTagName("iframe")).filter(t=>t.contentWindow===e.source)[0]}classifyZoneIds(e){const t=e.map(o.Nj).filter(o.yb);return{eligible:t.filter(o.xb).filter(o.pn),ineligible:t.filter(o.xj)}}applyExtraMarginBottom(t){if(t&&t.slot&&!t.isEmpty&&(t.creativeId||t.lineItemId||!t.isEmpty)){var s=t.slot.getSlotElementId();if(s){var o=document.getElementById(s);if(o)if((0,n.eq)(s)){var i=o?.closest(".js-zone-container");i.style.marginBottom="24px",e.cM("Applied extra margin to the bottom of "+s)}else e.cM("Not applying extra margin to the bottom of "+s+": shouldHaveReportButton = false");else e.cM("Not applying extra margin to the bottom of "+s+": resolved invalid adUnit element")}else e.cM("Not applying extra margin to the bottom of element: invalid adUnitElementId")}else e.cM("Not applying extra margin to the bottom of element: invalid SlotRenderEndedEvent")}async load(s=(0,n.kG)()){const a=this.opt.tlb_position===t.Above?["dfp-mlb","dfp-smlb"]:["dfp-mlb","dfp-smlb","dfp-tlb"];if(!this.isGptReady())return e.cM("Initializing..."),this.initGpt(),void googletag.cmd.push(()=>this.load(s));this.opt.adReportEnabled?(e.cM("Ad reporting enabled"),this.adReports=new i.t(googletag,this)):(e.cM("Ad reporting not enabled"),this.handleSlotRenderedNoAdReport()),this.opt.refresh?(e.cM("Ad refresh enabled"),this.adRefresh=new r(googletag,this.opt.refreshInterval)):e.cM("Ad refresh not enabled"),this.opt.brandSurveyEnabled&&(e.cM("Brand Survey enabled"),this.brandSurvey=new g(googletag,this.opt.brandSurveySettings)),e.cM("Attempting to load ads into ids: ",s);const{eligible:d,ineligible:l}=this.classifyZoneIds(s);if(this.initDebugPanel(googletag,d.concat(l)),d.forEach(e=>(0,o.cf)(e)),l.forEach(o.wo),0===d.length)return void e.cM("Found no ad ids on page");e.cM("Eligible ids:",d),this.opt.abd&&this.appendAdblockDetector();var c=googletag.pubads().getSlots();if(c){var p=c.filter(e=>s.indexOf(e.getSlotElementId())>=0);googletag.destroySlots(p)}this.opt.sf&&(googletag.pubads().setForceSafeFrame(!0),googletag.pubads().setSafeFrameConfig({allowOverlayExpansion:!0,allowPushExpansion:!0,sandbox:!0})),e.cM("Targeting consent: Checking...");let h=!1,m=!1;void 0!==this.opt.targeting_consent&&(m=!0,e.cM("Targeting consent: Parameter set"),e.cM("Targeting consent: Consent given? ",this.opt.targeting_consent),h=this.opt.targeting_consent),void 0!==this.opt.personalization_consent&&(e.cM("Personalization consent: Parameter set"),e.cM("Personalization consent: Consent given? ",this.opt.personalization_consent),h=h&&this.opt.personalization_consent),h=h&&m,this.setPrivacySettings(h),this.opt.ll||googletag.pubads().enableSingleRequest(),cam.sreEvent||(googletag.pubads().addEventListener("slotRenderEnded",e=>this.onSlotRendered(e)),cam.sreEvent=!0),await this.setTargeting();var u=d.filter(e=>!this.opt.ll||a.indexOf(e.id)<0),f=d.filter(e=>!!this.opt.ll&&a.indexOf(e.id)>=0);e.cM("Up front ids:",u),e.cM("Lazy loaded ids:",f),u.forEach(t=>{e.cM(`Defining ad for element ${t.id}`),this.defineSlot(t.id,googletag),t.setAttribute("data-dfp-zone","true")}),googletag.enableServices(),u.forEach(t=>{e.cM(`Displaying ad for element ${t.id}`),googletag.cmd.push(()=>googletag.display(t.id))}),this.opt.ll&&(e.cM("Enabling lazy loading for GAM"),googletag.pubads().enableLazyLoad({fetchMarginPercent:0,renderMarginPercent:0}),e.cM("Setting up lazy loaded ad units"),f.forEach(t=>{e.cM(`Lazy loading - Defining Slot ${t.id}`),this.defineSlot(t.id,googletag)}),f.forEach(t=>{e.cM(`Lazy loading - Displaying ad for element ${t.id}`),googletag.cmd.push(()=>googletag.display(t.id))}))}setPrivacySettings(e){e?googletag.pubads().setPrivacySettings({limitedAds:!1,nonPersonalizedAds:!1}):googletag.pubads().setPrivacySettings({limitedAds:!0,nonPersonalizedAds:!0})}async setTargeting(){if(!googletag)throw new Error("googletag not defined");let t=this.opt.targeting;if(!t)throw new Error("Targeting not defined (is "+typeof t+")");Object.keys(t).forEach(s=>{e.cM(`-> targeting - ${s}: ${t[s]}`),googletag.pubads().setTargeting(s,t[s])});let s=!1;if(void 0!==this.opt.targeting_consent&&(s=this.opt.targeting_consent),s){let t=(new c).getBrandSurveyEngagement();if(t.getTotalBrandImpressions().forEach((t,s)=>{e.cM(`-> targeting - BrandImpressions: ${s}: ${t}`),googletag.pubads().setTargeting("brand_"+s.toString()+"_impressions",t.toString())}),t.surveysIdsCompleted.forEach(t=>{e.cM(`-> targeting - SurveysTaken: ${t}`),googletag.pubads().setTargeting("survey_"+t+"_taken","true")}),this.clc_options.tgt_e&&this.getUserAccountTargetingPromise){let t=await this.getUserAccountTargetingPromise;t&&t.tgt_acct?(e.cM("-> targeting - User Account: "+t.tgt_acct),googletag.pubads().setTargeting("user-acct",t.tgt_acct)):e.cM("-> targeting - User Account: Not Found")}}}appendAdblockDetector(){const e=document.createElement("div");e.className="adsbox",e.id="clc-abd",e.style.position="absolute",e.style.pointerEvents="none",e.innerHTML="&nbsp;",document.body.appendChild(e)}onSlotRendered(s){try{const r=s.slot.getSlotElementId();let a=[];r||a.push("id=0");const d=document.getElementById(r);if(r&&!d&&a.push("el=0"),0!==a.length)return void this.stalled(a.join("&"));const{path:l,sizes:c,zone:g}=(0,n.Z7)(r);if(this.collapsed[g]&&s.isEmpty)return e.cM(`No line item for the element #${d.id}... collapsing.`),void(0,o.wo)(d);if(this.slotsRenderedEvents.push(s),s.lineItemId||s.creativeId||!s.isEmpty){e.cM(`Rendered ad for element #${d.id} [line item #${s.lineItemId}]`),e.cM(s);var i=d.parentElement;if(i.classList.contains("js-zone-container")){switch((0,o.cf)(i),r){case"dfp-tlb":this.opt.tlb_position===t.Above?i.classList.add("mb8"):i.classList.add("mt16");break;case"dfp-tag":i.classList.add("mb8");break;case"dfp-msb":i.classList.add("mt16");break;case"dfp-mlb":case"dfp-smlb":case"dfp-bmlb":i.classList.add("my8");break;case"dfp-isb":i.classList.add("mt24");break;case"dfp-m-aq":i.classList.add("my12"),i.classList.add("mx-auto")}(0,o.$Z)(i),(0,o.$Z)(d)}else e.cM(`No ad for element #${d.id}, collapsing`),e.cM(s),(0,o.wo)(d)}}catch(t){e.cM("Exception thrown onSlotRendered"),e.cM(t),this.stalled("e=1")}}stalled(e){(new Image).src=`https://${this.clc_options.h}/stalled.gif?${e}`}defineSlot(t,s){"dfp-isb"===t&&(e.cM("-> targeting - Sidebar: Inline"),s.pubads().setTargeting("Sidebar",["Inline"])),"dfp-tsb"===t&&(e.cM("-> targeting - Sidebar: Right"),s.pubads().setTargeting("Sidebar",["Right"]));const{path:o,sizes:i,zone:r}=(0,n.Z7)(t);e.cM(`Defining slot for ${t}: ${o}, sizes: ${JSON.stringify(i)}`),s.defineSlot(o,i,t).addService(s.pubads())}importGptLibrary(){this.gptImported||(this.gptImported=!0,void 0===this.opt.targeting_consent||this.opt.targeting_consent?(0,o.Gx)("https://securepubads.g.doubleclick.net/tag/js/gpt.js"):(0,o.Gx)("https://pagead2.googlesyndication.com/tag/js/gpt.js"))}isGptReady(){return"undefined"!=typeof googletag&&!!googletag.apiReady}initGpt(){"undefined"==typeof googletag&&(window.googletag={cmd:(0,o.QZ)(()=>this.importGptLibrary())})}getUserAccountTargeting(){if(this.clc_options.tgt_e&&this.clc_options.tgt_p>0){if(e.cM("Targeting enabled."),this.clc_options.tgt_p<100){e.cM("Targeting rate limit enabled. Rolling the dice...");const t=Math.floor(100*Math.random())+1;if(e.cM("Rolled "+t+" and the max is "+this.clc_options.tgt_p),t>this.clc_options.tgt_p)return void e.cM("Will not request targeting.")}return e.cM("Will request targeting."),function(e,t,s){if(e){const e=new Headers;return e.append("Accept","application/json"),async function(e,t={},s=5e3){if("number"!=typeof s&&null!=s&&!1!==s){if("string"!=typeof s)throw new Error("fetchWithTimeout: timeout must be a number");if(s=parseInt(s),isNaN(s))throw new Error("fetchWithTimeout: timeout must be a number (or string that can be parsed to a number)")}const n=new AbortController,{signal:o}=n,i=fetch(e,{...t,signal:o}),r=setTimeout(()=>n.abort(),s);try{const e=await i;return clearTimeout(r),e}catch(e){throw clearTimeout(r),e}}(t,{method:"GET",mode:"cors",headers:e},s).then(e=>e.json())}return Promise.reject("No consent")}(this.opt.targeting_consent,this.clc_options.tgt_u,this.clc_options.tgt_to).catch(t=>{e.vU("Error fetching user account targeting"),e.vU(t)})}e.cM("Targeting disabled. Will not request account targeting data.")}initDebugPanel(t,s){e.cM("initDebugPanel"),e.cM("Not showing debug panel")}},window.clcGamLoaderOptions&&(cam.init(),cam.load())})()})();</script>
<footer id="footer" class="site-footer js-footer theme-light__forced" role="contentinfo">
<div class="site-footer--container">
<div class="site-footer--logo">
<a href="https://stackoverflow.com" aria-label="Stack Overflow"><svg aria-hidden="true" class="native svg-icon iconLogoGlyphMd" width="32" height="37" viewBox="0 0 32 37"><path d="M26 33v-9h4v13H0V24h4v9h22Z" fill="#BCBBBB"/><path d="m21.5 0-2.7 2 9.9 13.3 2.7-2L21.5 0ZM26 18.4 13.3 7.8l2.1-2.5 12.7 10.6-2.1 2.5ZM9.1 15.2l15 7 1.4-3-15-7-1.4 3Zm14 10.79.68-2.95-16.1-3.35L7 23l16.1 2.99ZM23 30H7v-3h16v3Z" fill="#F48024"/></svg></a>
</div>
<nav class="site-footer--nav">
<div class="site-footer--col">
<h5 class="-title"><a href="https://stackoverflow.com" class="js-gps-track" data-gps-track="footer.click({ location: 2, link: 15})">Stack Overflow</a></h5>
<ul class="-list js-primary-footer-links">
<li><a href="/questions" class="js-gps-track -link" data-gps-track="footer.click({ location: 2, link: 16})">Questions</a></li>
<li><a href="/help" class="js-gps-track -link" data-gps-track="footer.click({ location: 2, link: 3 })">Help</a></li>
</ul>
</div>
<div class="site-footer--col">
<h5 class="-title"><a href="https://stackoverflow.co/" class="js-gps-track" data-gps-track="footer.click({ location: 2, link: 19 })">Products</a></h5>
<ul class="-list">
<li><a href="https://stackoverflow.co/teams/" class="js-gps-track -link"
data-ga="[&quot;teams traffic&quot;,&quot;footer - site nav&quot;,&quot;stackoverflow.com/teams&quot;,null,{&quot;dimension4&quot;:&quot;teams&quot;}]"
data-gps-track="footer.click({ location: 2, link: 29 })">Teams</a></li>
<li><a href="https://stackoverflow.co/advertising/" class="js-gps-track -link" data-gps-track="footer.click({ location: 2, link: 21 })">Advertising</a></li>
<li><a href="https://stackoverflow.co/collectives/" class="js-gps-track -link" data-gps-track="footer.click({ location: 2, link: 40 })">Collectives</a></li>
<li><a href="https://stackoverflow.co/talent/" class="js-gps-track -link" data-gps-track="footer.click({ location: 2, link: 20 })">Talent</a></li>
</ul>
</div>
<div class="site-footer--col">
<h5 class="-title"><a class="js-gps-track" data-gps-track="footer.click({ location: 2, link: 1 })" href="https://stackoverflow.co/">Company</a></h5>
<ul class="-list">
<li><a class="js-gps-track -link" data-gps-track="footer.click({ location: 2, link: 1 })" href="https://stackoverflow.co/">About</a></li>
<li><a class="js-gps-track -link" data-gps-track="footer.click({ location: 2, link: 27 })" href="https://stackoverflow.co/company/press/">Press</a></li>
<li><a class="js-gps-track -link" data-gps-track="footer.click({ location: 2, link: 9 })" href="https://stackoverflow.co/company/work-here/">Work Here</a></li>
<li><a class="js-gps-track -link" data-gps-track="footer.click({ location: 2, link: 7 })" href="https://stackoverflow.com/legal">Legal</a></li>
<li><a class="js-gps-track -link" data-gps-track="footer.click({ location: 2, link: 8 })" href="https://stackoverflow.com/legal/privacy-policy">Privacy Policy</a></li>
<li><a class="js-gps-track -link" data-gps-track="footer.click({ location: 2, link: 37 })" href="https://stackoverflow.com/legal/terms-of-service/public">Terms of Service</a></li>
<li><a class="js-gps-track -link" data-gps-track="footer.click({ location: 2, link: 13 })" href="https://stackoverflow.co/company/contact/">Contact Us</a></li>
<li class="" id="consent-footer-link"><a class="js-gps-track -link js-cookie-settings" data-gps-track="footer.click({ location: 2, link: 38 })" href="#" data-consent-popup-loader="footer">Cookie Settings</a></li>
<li><a class="js-gps-track -link" data-gps-track="footer.click({ location: 2, link: 39 })" href="https://stackoverflow.com/legal/cookie-policy">Cookie Policy</a></li>
</ul>
</div>
<div class="site-footer--col site-footer--categories-nav">
<div>
<h5 class="-title"><a href="https://stackexchange.com" data-gps-track="footer.click({ location: 2, link: 30 })">Stack Exchange Network</a></h5>
<ul class="-list">
<li>
<a href="https://stackexchange.com/sites#technology" class="-link js-gps-track" data-gps-track="footer.click({ location: 2, link: 24 })">
Technology
</a>
</li>
<li>
<a href="https://stackexchange.com/sites#culturerecreation" class="-link js-gps-track" data-gps-track="footer.click({ location: 2, link: 24 })">
Culture &amp; recreation
</a>
</li>
<li>
<a href="https://stackexchange.com/sites#lifearts" class="-link js-gps-track" data-gps-track="footer.click({ location: 2, link: 24 })">
Life &amp; arts
</a>
</li>
<li>
<a href="https://stackexchange.com/sites#science" class="-link js-gps-track" data-gps-track="footer.click({ location: 2, link: 24 })">
Science
</a>
</li>
<li>
<a href="https://stackexchange.com/sites#professional" class="-link js-gps-track" data-gps-track="footer.click({ location: 2, link: 24 })">
Professional
</a>
</li>
<li>
<a href="https://stackexchange.com/sites#business" class="-link js-gps-track" data-gps-track="footer.click({ location: 2, link: 24 })">
Business
</a>
</li>
<li class="mt16 md:mt0">
<a href="https://api.stackexchange.com/" class="-link js-gps-track" data-gps-track="footer.click({ location: 2, link: 24 })">
API
</a>
</li>
<li>
<a href="https://data.stackexchange.com/" class="-link js-gps-track" data-gps-track="footer.click({ location: 2, link: 24 })">
Data
</a>
</li>
</ul>
</div>
</div>
</nav>
<div class="site-footer--copyright fs-fine md:mt24">
<ul class="-list -social md:mb8">
<li><a class="js-gps-track -link" data-gps-track="footer.click({ location: 2, link:4 })" href="https://stackoverflow.blog?blb=1">Blog</a></li>
<li><a href="https://www.facebook.com/officialstackoverflow/" class="-link js-gps-track" data-gps-track="footer.click({ location: 2, link: 31 })">Facebook</a></li>
<li><a href="https://twitter.com/stackoverflow" class="-link js-gps-track" data-gps-track="footer.click({ location: 2, link: 32 })">Twitter</a></li>
<li><a href="https://linkedin.com/company/stack-overflow" class="-link js-gps-track" data-gps-track="footer.click({ location: 2, link: 33 })">LinkedIn</a></li>
<li><a href="https://www.instagram.com/thestackoverflow" class="-link js-gps-track" data-gps-track="footer.click({ location: 2, link: 36 })">Instagram</a></li>
</ul>
<p class="md:mb0">
Site design / logo &#169; 2023 Stack Exchange Inc; user contributions licensed under <span class='td-underline'><a href="https://stackoverflow.com/help/licensing">CC BY-SA</a></span>. <span id="svnrev">rev&nbsp;2023.11.30.1842</span>
</p>
</div>
</div>
</footer>
<!-- Google tag (gtag.js) -->
<script async src="https://www.googletagmanager.com/gtag/js?id=G-WCZ03SZFCQ"></script>
<script>
window.dataLayer = window.dataLayer || [];
function gtag() { dataLayer.push(arguments); }
</script>
<script>
StackExchange.ready(function() {
var ga3Settings = {
autoLink: ["stackoverflow.blog","info.stackoverflowsolutions.com","stackoverflowsolutions.com"],
sendTitles: true,
tracker: window.ga,
trackingCodes: [
'UA-108242619-1'
],
checkDimension: 'dimension42'
};
var customGA4Dimensions = {};
customGA4Dimensions["routename"] = "Questions/Show";
customGA4Dimensions["post_id"] = "20626994";
customGA4Dimensions["tags"] = "|python|scipy|normal-distribution|";
var ga4Settings = {
tracker: gtag,
trackingCodes: [
'G-WCZ03SZFCQ'
],
consentsToPerformanceCookies: "denied",
consentsToTargetingCookies: "denied",
eventParameters: customGA4Dimensions,
checkForAdBlock: true,
sendTitles: true,
trackClicks: false,
};
StackExchange.ga.init({ GA3: ga3Settings, GA4: ga4Settings });
StackExchange.ga.setDimension('dimension2', '|python|scipy|normal-distribution|');
StackExchange.ga.setDimension('dimension3', 'Questions/Show');
StackExchange.ga.setDimension('dimension7', "1701628369.1770797741");
StackExchange.ga.trackPageView();
});
</script>
<div class="ff-sans ps-fixed z-nav-fixed ws4 sm:w-auto p32 sm:p16 bg-black-600 fc-white bar-lg b16 l16 r16 js-consent-banner">
<svg aria-hidden="true" class="mln4 mb24 sm:d-none svg-spot spotCookieLg" style="color: var(--theme-button-primary-background-color, var(--theme-primary-200))" width="96" height="96" viewBox="0 0 96 96">
<path d="M35 45.5a7.5 7.5 0 11-15 0 7.5 7.5 0 0115 0zM63.5 63a7.5 7.5 0 100-15 7.5 7.5 0 000 15zm-19 19a7.5 7.5 0 100-15 7.5 7.5 0 000 15z" opacity=".2" />
<path d="M56.99 2.53a23.1 23.1 0 0114.66 6.15h.01l.01.02c.57.55.61 1.27.5 1.74v.07a10.95 10.95 0 01-3.07 4.77 9 9 0 01-6.9 2.5 10.34 10.34 0 01-9.72-10.44v-.08a10 10 0 011.03-3.74l.01-.03.02-.02c.28-.5.82-.92 1.52-.95.63-.02 1.27-.02 1.93.01zm12.04 7.83a20.1 20.1 0 00-12.2-4.83l-.92-.03c-.23.6-.38 1.25-.43 1.94a7.34 7.34 0 006.95 7.34 6 6 0 004.64-1.7c.94-.88 1.6-1.9 1.96-2.72zm15.3 8.76a6.84 6.84 0 00-5.09-.24 7.9 7.9 0 00-3.28 2.05 1.8 1.8 0 00-.3 1.95l.02.02v.02a15.16 15.16 0 008.74 7.47c.64.23 1.32.08 1.8-.33a6.63 6.63 0 001.63-1.97l.01-.03.01-.03c1.67-3.5-.12-7.32-3.54-8.91zm-5.5 3.28c.36-.25.82-.5 1.35-.67.92-.3 1.92-.35 2.89.1 2.14 1 2.92 3.14 2.11 4.88-.12.21-.26.41-.43.6l-.26-.1a12.29 12.29 0 01-5.66-4.81zM32 24a2 2 0 11-4 0 2 2 0 014 0zm12 21a2 2 0 11-4 0 2 2 0 014 0zm36 4a2 2 0 11-4 0 2 2 0 014 0zm-7 21a2 2 0 11-4 0 2 2 0 014 0zM59 81a2 2 0 11-4 0 2 2 0 014 0zM22 63a2 2 0 11-4 0 2 2 0 014 0zm27 7a9 9 0 11-18 0 9 9 0 0118 0zm-3 0a6 6 0 10-12 0 6 6 0 0012 0zM33 41a9 9 0 11-18 0 9 9 0 0118 0zm-15 0a6 6 0 1012 0 6 6 0 00-12 0zm50 11a9 9 0 11-18 0 9 9 0 0118 0zm-3 0a6 6 0 10-12 0 6 6 0 0012 0zM44.08 4.24c.31.48.33 1.09.05 1.58a17.46 17.46 0 00-2.36 8.8c0 9.55 7.58 17.24 16.85 17.24 2.97 0 5.75-.78 8.16-2.15a1.5 1.5 0 012.1.66 12.08 12.08 0 0011 6.74 12.4 12.4 0 007.85-2.75 1.5 1.5 0 012.38.74A45.76 45.76 0 0192 48.16c0 24.77-19.67 44.9-44 44.9S4 72.93 4 48.16C4 25.23 20.84 6.28 42.64 3.58a1.5 1.5 0 011.44.66zM40.22 7C21.32 10.71 7 27.7 7 48.16c0 23.17 18.39 41.9 41 41.9s41-18.73 41-41.9c0-3.52-.42-6.93-1.22-10.2a15.5 15.5 0 01-7.9 2.15c-5.5 0-10.36-2.83-12.97-7.1a19.46 19.46 0 01-8.28 1.85c-11 0-19.86-9.1-19.86-20.24 0-2.7.52-5.26 1.45-7.62zM92 91a2 2 0 100-4 2 2 0 000 4zM7 8.5a2.5 2.5 0 11-5 0 2.5 2.5 0 015 0zM82.5 90a1.5 1.5 0 100-3 1.5 1.5 0 000 3zm9.5-7.5a1.5 1.5 0 11-3 0 1.5 1.5 0 013 0zM13.5 8a1.5 1.5 0 100-3 1.5 1.5 0 000 3zM80 14.5a1.5 1.5 0 11-3 0 1.5 1.5 0 013 0zM53.5 20a1.5 1.5 0 100-3 1.5 1.5 0 000 3z" />
</svg>
<p class="fs-body2 fw-bold mb4">
Your privacy
</p>
<p class="mb16 s-anchors s-anchors__inherit s-anchors__underlined">
By clicking “Accept all cookies”, you agree Stack Exchange can store cookies on your device and disclose information in accordance with our <a href="https://stackoverflow.com/legal/cookie-policy">Cookie Policy</a>.
</p>
<div class="d-flex gs8 sm:fd-column">
<button class="flex--item6 s-btn s-btn__filled js-accept-cookies js-consent-banner-hide">
Accept all cookies
</button>
<button class="flex--item6 s-btn s-btn__filled js-reject-cookies js-consent-banner-hide">
Necessary cookies only
</button>
</div>
<div class="d-flex mt8 sm:fd-column">
<button class="flex--item12 s-btn s-btn__filled js-cookie-settings" data-consent-popup-loader="banner">
Customize settings
</button>
</div>
</div>
<div id="onetrust-consent-sdk" class="d-none"></div>
<div id="onetrust-banner-sdk" data-controller="s-modal"></div>
<div id="ot-pc-content" class="d-none"></div>
<div id="onetrust-style" class="d-none">&nbsp;</div>
<div class="d-none js-consent-banner-version" data-consent-banner-version="1"></div>
</body>
</html>